Vous êtes sur la page 1sur 64

Ibpsprepadda is an initiative by Bankers & students.

Our endeavor is to help students in preparation of Banking exams like


SBI, IBPS, RBI, NICL, NIACL, LIC etc. Our endeavor is to provide all the relevant study materials, reference materials etc,
mainly free of cost or on little cost. We are committed to serve you better. Please visit our website & Facebook page daily to
get benefited. Facebook- www.facebook.com/ibpsprepadda

SBI PO PRELIMS- TEST-1


Directions:
1.This Sample paper contains three sections as follows :
Test I - English Language Q.Nos. 130
Test II- Quantitative aptitude 3165
Test III- Reasoning Q.Nos. 66100
2. You will be given an aggregate time of 1:00 hours to
answer all the three tests. The tests are not separately
timed. You may distribute the time as you wish but
remember that to qualify in the written test as a whole
you have to qualify on each of the three tests separately.
3.There will be penalty for wrong answers marked by you.
For each question for which a wrong answer has been
given by you, one-fourth or 0.25 of the marks assigned
to that question will be deducted as penalty.

All the best!

Ibpsprepadda is an initiative by Bankers & students. Our endeavor is to help students in preparation of Banking exams like
SBI, IBPS, RBI, NICL, NIACL, LIC etc. Our endeavor is to provide all the relevant study materials, reference materials etc,
mainly free of cost or on little cost. We are committed to serve you better. Please visit our website & Facebook page daily to
get benefited. Facebook- www.facebook.com/ibpsprepadda

Direction (1-10):
Read the following passage carefully and answer the questions given below it. Certain word/
phrases are printed in bold to help you locate them while answering some of the questions.
Better late than never. Given the pace at which pollution has been rising plus its pronounced
health and economic costs, India should have had a National Air Quality Index (AQI) up and
running many years ago. Still, there`s no denying a sigh of relief that this index was finally
launched yesterday at a state environment ministers conference to begin with it will monitor
pollution levels in 10 major cities.
Failure of government apparatus to manage particulate matter built up from industry, transport
and environmental factors has seen ambient air quality declining perilously throughout urban
India. Latest data analysis finds that in the capital, for example, air quality was poor or worse for
86% of this winter. Actually, the 7% of days when it was very poor would have triggered
emergency health warnings under the US national index. In other words, with real time
information on air quality citizens can be empowered to take preventive actions like keeping
vulnerable toddlers indoors or postponing picnic plans or going to work by public transport
rather than the bike. And as AQI gathers data and strength, it can also put pressure on high
polluters to clean up their act.
In launching this index, Prime Minister Narendra Modi placed it in the larger context of climate
change. He is absolutely right that developed nations do not have the right to lecture India
given their far worse records on climate change India is, after all, one of the lowest per capita
emitters of greenhouse gases but you won`t often find this mentioned in western descriptions of
Indias role. But for that very reason some of the lifestyle changes mooted by Modi such as
switching off lights on full moon nights or observing cycle Sundays arent practical and can
tackle the problem only at the margins.
Surely the problem lies elsewhere on the massive government failures that have brought
India and Indians to this impasse of a toxic urban environment. Underperformance of
2

Ibpsprepadda is an initiative by Bankers & students. Our endeavor is to help students in preparation of Banking exams like
SBI, IBPS, RBI, NICL, NIACL, LIC etc. Our endeavor is to provide all the relevant study materials, reference materials etc,
mainly free of cost or on little cost. We are committed to serve you better. Please visit our website & Facebook page daily to
get benefited. Facebook- www.facebook.com/ibpsprepadda

environmental regulators has clearly been shocking. Policies across fuel, transport, power,
industry and other sectors have bungled the clean air objective. Government should put its own
house in order rather than pass the onus of change onto people. It also needs to come up with
technological solutions rather than simply refer to past traditions. Stop fetishising brooms
which raise dust rather than clean air in the era of vacuum cleaners.

1. Which of the following word has similar meaning to apparatus?


a. Framework
b. connection
c. Approach
d.willingness
2. Which of the following word has similar meaning to mooted?
a. Opposed
b.favour
c.raise
d.flaunt
3.

What is the reason behind declining air quality in urban area ?


a. Government is not focused on urban area AQI
b. Industries in Urban areas are emitting carbon gases
c. Failure of government mechanism.
d. Because, there is no quality index in India.

4.

Switching off lights on full moon nights or observing cycle Sundays can lead to?
a. Improved Index
b. US ranking
c. Insignificant help in controlling climate change issue.
d.AQI US national index.

Ibpsprepadda is an initiative by Bankers & students. Our endeavor is to help students in preparation of Banking exams like
SBI, IBPS, RBI, NICL, NIACL, LIC etc. Our endeavor is to provide all the relevant study materials, reference materials etc,
mainly free of cost or on little cost. We are committed to serve you better. Please visit our website & Facebook page daily to
get benefited. Facebook- www.facebook.com/ibpsprepadda

5.

What is the tone of the passage?


a. Ironic
b.Biased
c.Cynical
d.Apologetic

6.
What it means in passage Government should put its own house in order rather than pass the
onus of change onto people
a. Stop blaming folks, Improve your policies.
b. Government should take care of their own houses rather than making policies.
c. Provide subsidies on green initiatives.
d. Impose fine on industries.
7.

How people can be benefited by AQI?


a. By implanting it people will get immediate fresh Air.
b. By implanting it industries will not evacuate harmful gases.

c. Preventive actions like keeping vulnerable toddlers indoors or postponing picnic plans or
going to work by public transport.
d. They will feel free of toxicities.
8.
Why author insisted that India should have had a National Air Quality Index (AQI) up and
running many years ago.
a. India lost international rank because of it.
b. Nation paid huge on health services, lost economic development.
c. Cancer and other critical disease could have been lower rate of speediness.
d. Our parents could have live quality life.
Directions (Q. 9 -10): Choose the word/group of words which is MOST SIMILAR in meaning to the
word/group of words printed in bold as used in the passage.
9. Spiked
1) projected 2) peaked 3) declined 4) pierced 5) developed

Ibpsprepadda is an initiative by Bankers & students. Our endeavor is to help students in preparation of Banking exams like
SBI, IBPS, RBI, NICL, NIACL, LIC etc. Our endeavor is to provide all the relevant study materials, reference materials etc,
mainly free of cost or on little cost. We are committed to serve you better. Please visit our website & Facebook page daily to
get benefited. Facebook- www.facebook.com/ibpsprepadda

10. Soaring
1) dropping 2) slumping 3) escalating 4) bitter 5) cutting
Directions (Q. 11-15): Read each sentence to find out whether there is any grammatical or
idiomatic error in it. The
error, if any, will be in one part of the sentence. The number of that part is the answer. If there is
no error, the answer is (5).
(Ignore errors of punctuation, if any.)
11. (1) Besides his blue turban / (2) and flowing beard / (3) there is almost nothing / (4) to distinguish him
from other
residents. / (5) No error
12. (1) The main reason / (2) for leaving the volatile region / (3) was the Talibans / (4) raise in
prominence. / (5) No error
13. (1) Strangers who I met at the stadium / (2) took me out for dinner / (3) and sightseeing, without letting
me / (4) pay a
penny. / (5) No error
14. (1) Although I am a sports buff, / (2) I feel we should not unnecessarily / (3) keep up the pretence of
sporting ties / (4)
when all is not good between us. / (5) No error
15. (1) This is equivalent of / (2) a woman employee getting / (3) five years of paid leave / (4) in a 30-year
working span. / (5)

Directions for questions 16 to 25: In the following passage, there are blanks, each of which has been
numbered. These numbers are printed below the passage and against each, five words are suggested,
one of which fits the blank appropriately. Find out then appropriate word in each case.

Greenhouse gases are only (16) of the story when it comes to global warming. Changes to one
part of the climate system can (17) additional changes to the way the planet absorbs or reflects
energy. These secondary changes are (18) climate feedbacks, and they could more than double
the amount of warmingcaused by carbon dioxide alone. The primary feedbacks are (19) to snow
and ice, water vapour,clouds and the carbon cycle.Perhaps the most well (20) feedback comes
frommelting snow and ice in the Northern Hemisphere.Warming temperatures are already (21) a
growing percentage of Artic sea ice, exposing dark ocean water during the (22) sunlight of
summer. Snow cover on land is also (23) in many areas. In the (24) of snow and ice, these areas
go from having right, sunlight reflecting surfaces that cool the planet to having dark, sunlight
absorbing surfaces that (25) more energy into the Earth system and cause more warming.
16.(1) whole (2) part
(3) material (4) issue
(5) most
5

Ibpsprepadda is an initiative by Bankers & students. Our endeavor is to help students in preparation of Banking exams like
SBI, IBPS, RBI, NICL, NIACL, LIC etc. Our endeavor is to provide all the relevant study materials, reference materials etc,
mainly free of cost or on little cost. We are committed to serve you better. Please visit our website & Facebook page daily to
get benefited. Facebook- www.facebook.com/ibpsprepadda

17.(1) raise (2) brings (3) refer


(4) stop (5) cause
18.(1) sensed (2) called (3) nothing
(4) but (5) term
19.(1) due (2) results (3) reason
(4) those (5) because
20.(1) done (2) known (3) ruled
(4) bestowed (5) said
21.(1) mastering (2) sending (3) melting
(4) calming (5) increasing
22.(1) makeshift (2) ceasing
(3) troubled (4) perpetual (5) absent
23.(1) dwindling (2) manufactured
(3) descending (4) generating
(5) supplied
24.(1) progress (2) reduced
(3) existence (4) midst
(5) absence
25.(1) repel (2) waft (3) monitor
(4) bring (5) access
Directions (Q 26-30): Rearrange the following six sentences (A), (B), (C), (D), (E) and (F) in the
proper sequence to form a meaningful paragraph and then answer the questions given below.

(A) Being rejected from almost every management and banking competition I applied for, taught me
that strategy comes
from planning and real world application of ideas not seat of the pants thinking.
6

Ibpsprepadda is an initiative by Bankers & students. Our endeavor is to help students in preparation of Banking exams like
SBI, IBPS, RBI, NICL, NIACL, LIC etc. Our endeavor is to provide all the relevant study materials, reference materials etc,
mainly free of cost or on little cost. We are committed to serve you better. Please visit our website & Facebook page daily to
get benefited. Facebook- www.facebook.com/ibpsprepadda

(B) My failure taught me to be clearer about my practice, vision, and objectives when persuading others
to get behind me.
(C) Getting dinged from 4 out of the 9 top banks taught me that accuracy matters just as much as speed
when evaluating
what organization or post you should align yourself with.
(D) Getting out from second round examinations taught me that I should funnel my desire for accuracy
off stage.
(E) Failing my exam taught me that you have to be prepared, always, in order to be successful.
F) I've chosen a field that requires me to think logically about time management problems and apply my
interest in the
quantitative aptitude and accuracy.

26. Which of the following would be the FOURTH sentence after rearrangement?
1) D 2) E 3) A 4) C 5) F
27. Which of the following would be the LAST sentence after rearrangement?
1) A 2) E 3) B 4) C 5) F
28. Which of the following would be the FIFTH sentence after rearrangement?
1) A 2) E 3) C 4) F 5) D
29. Which of the following would be the FIRST sentence after rearrangement?
1) F 2) C 3) E 4) A 5) B
30. Which of the following would be the THIRD sentence after rearrangement?
1) D 2) C 3) F 4) A 5) E

Ibpsprepadda is an initiative by Bankers & students. Our endeavor is to help students in preparation of Banking exams like
SBI, IBPS, RBI, NICL, NIACL, LIC etc. Our endeavor is to provide all the relevant study materials, reference materials etc,
mainly free of cost or on little cost. We are committed to serve you better. Please visit our website & Facebook page daily to
get benefited. Facebook- www.facebook.com/ibpsprepadda

Test II
Quantitative Aptitude

31. 7225 1/5+ (45)2 =?


1) 2058 2) 2042 3) 2040 4) 2038 5) None of these

32. 6.25 0.5 ?10


1)5 2) 500 3) 50 4) 2.5 5) None of these

33. 3 1216716384= ?11.5


1)204 2) 234 3) 286 4) 256 5) None of these

34. 1500, 1581, 1664, 1749, 1836, ?, 2016


1) 1912 2) 1919 3) 1932 4) 1925 5) None of these
35. 6, 4, 5.5, 10.25, 23.0, ?
1) 62 2) 42.5 3) 49 4) 56.25 5) None of these
36. 19, 68, 104, 129, 145, ?
1) 151 2) 157 3) 154 4) 161 5) None of these
37.The circumference of a circle exceeds its diameter by 16.8 cm. Find the circumference of the circle ?
1) 24.6 cm 2) 246.12 cm 3) 12.67 cm 4) 32.33 cm 5) None of these
38. Find out the probability of forming 187 or 215 with the digits 1, 2, 3, 4, 5, 6, 7, 8, 9 when only
numbers of 3 digits are formed and when repetitions are not allowed.
1) 25/5042) 7/504 3) 1/252 4) 13/252 5) None of these

Ibpsprepadda is an initiative by Bankers & students. Our endeavor is to help students in preparation of Banking exams like
SBI, IBPS, RBI, NICL, NIACL, LIC etc. Our endeavor is to provide all the relevant study materials, reference materials etc,
mainly free of cost or on little cost. We are committed to serve you better. Please visit our website & Facebook page daily to
get benefited. Facebook- www.facebook.com/ibpsprepadda

Directions (Q. 39 -43 ) : Study the table carefully to answer the questions that follow:
Percentage of students in MBA class, who sought employment in the area of Finance, Marketing
and Software :

Ibpsprepadda is an initiative by Bankers & students. Our endeavor is to help students in preparation of Banking exams like
SBI, IBPS, RBI, NICL, NIACL, LIC etc. Our endeavor is to provide all the relevant study materials, reference materials etc,
mainly free of cost or on little cost. We are committed to serve you better. Please visit our website & Facebook page daily to
get benefited. Facebook- www.facebook.com/ibpsprepadda

39. In 2007, students seeking jobs in finance earned more than those opting for software (per annum)
by
1) ` 1.764 lakhs 2) ` 17.64 lakh 3) ` 21.168 lakh 4) ` 12 lakh 5) None of these
40.What is the percentage increase in the average monthly salary offered to a software student over the
given five years ?
10

Ibpsprepadda is an initiative by Bankers & students. Our endeavor is to help students in preparation of Banking exams like
SBI, IBPS, RBI, NICL, NIACL, LIC etc. Our endeavor is to provide all the relevant study materials, reference materials etc,
mainly free of cost or on little cost. We are committed to serve you better. Please visit our website & Facebook page daily to
get benefited. Facebook- www.facebook.com/ibpsprepadda

1) 70 2) 41 3) 67 4) 43 5) None of these
41. The number of students, who get jobs in finance is less than the students getting software Jobs, in
the five years by
1) 4 2) 42 3) 9 4) 226 5) None of these
42.What is the percentage increase in the average salary of finance from 20042008 ?
1) 26% 2) 29% 3) 36% 4) 32% 5) None of these
43.. The average annual rate at which the initial salary offered in marketing increase is
1) 31.5 % 2) 22.4 % 3) 71.32 % 4) 29 % 5) None of these

Directions (Q. 44 -48) : Study the graph carefully to answer the questions that follow :
Profit (In ` lakhs) made by 3 companies over the years :

44.What is the percent increase in profit made by company A over the years ?
11

Ibpsprepadda is an initiative by Bankers & students. Our endeavor is to help students in preparation of Banking exams like
SBI, IBPS, RBI, NICL, NIACL, LIC etc. Our endeavor is to provide all the relevant study materials, reference materials etc,
mainly free of cost or on little cost. We are committed to serve you better. Please visit our website & Facebook page daily to
get benefited. Facebook- www.facebook.com/ibpsprepadda

1) 150 2) 60 3) 120 4) 82 5) None of these


45. Profit made by company C in the year 2009 was what percent of total profit made by all the three
companies in that year?
1) 111 2)151 1/7 3) 46 2/7 4) 42 6/7 5) None of these
46. If the income of company B in the year 2010 was ` 14 lakhs, what was its expenditure in that year ?
1) ` 20 lakh 2) ` 13 lakh 3) ` 16 lakh 4) ` 7 lakh 5) None of these
47.What is the percent increase in profit of company A in the year 2009 from the previous year ?
1) 50 2) 33 1/3 3) 66 1/3 4) 42 5) None of these

48. If the expenditure of company C in the year 2013 was ` 14.44 lakhs, what was its income in that
year?
1) ` 22.44 lakh 2) ` 17.5 lakh 3) ` 16.22 lakh 4) Can't be determined 5) None of these
49. A and B are two stations. A train goes from A to B at 64 km/hr and returns to A at a slower speed. If
its average speed for
the whole journey is 56 km/hr, at what speed did it return ?
1) 47.46 km/hr 2) 49.77 km/hr 3) 42 km/hr 4) 38 km/hr 5) None of these
50. A bag was sold at a profit of 10%. If its cost price was 5% less and it was sold for ` 7 more, the gain
would have been 20%.
Find the cost price of the bag ?
1)` 175 2) ` 225 3) ` 182.50 4) ` 205 5) None of these
51.In how many ways can the letters of the word EQUATION be arranged so that all the vowels come
together ?
9

1) 4!5! 2) 9! 4! 5! 3)

C 4 . 9C5 4 ) ! 9! 5) None of these

52.A speaks the truth 3 out of 4 times and B 5 out of 6 times. What is probability that they will
contradict each other in stating the same fact ?
1) 5/6

2) 2/3 3) 1/6 4) 1/5 5) None of these


12

Ibpsprepadda is an initiative by Bankers & students. Our endeavor is to help students in preparation of Banking exams like
SBI, IBPS, RBI, NICL, NIACL, LIC etc. Our endeavor is to provide all the relevant study materials, reference materials etc,
mainly free of cost or on little cost. We are committed to serve you better. Please visit our website & Facebook page daily to
get benefited. Facebook- www.facebook.com/ibpsprepadda

53.A and B can do a piece of work in 45 and 40 days respectively. They began the work together, but A
leaves after some days and B finished the remaining work in 23 days. After how many days did A leave ?
1) 11 days 2) 9 days 3) 8 days 4) 13 days 5) None of these
Directions (Q. 54-58): Each question below is followed by two statements A and B. You have to decide
whether the data given in the statements are sufficient for answering the questions. Read both the
statement and give answer :
1) If the statement A alone is sufficient to answer the question but the statement B alone is not
sufficient.
2) If the statement B alone is sufficient to answer the question but the statement A alone is not
sufficient.
3) If both statements A and B together are needed to answer the question.
4) If either the statement A alone or statement B alone is sufficient to answer the question.
5) If the data even in both statements A and B together are not sufficient to answer the question.
54. What is the profit earned by selling a pen for ` 120 ?
A) The cost price of 5 such pens is equal to selling price of 4 such pens.
B) 25% profit is earned by selling each pen.
55. What is the salary of C, in a group of A, B, C, D and E, whose average salary is `12000 ?
A) C's salary is 2.5 times B's salary.
B) Average salary of A and B is ` 4000.
56. What is the area of the circle ?
A) The breadth of a rectangle is the radius of circle.
B) The radius of the circle is equal to the side of a square of area 169 sq m.
57. What is the three digit number ?
A) 3/5 of a number is less by 90 of that number.
B) of a number is 25% of that number.
58. What is the speed of the car ?
13

Ibpsprepadda is an initiative by Bankers & students. Our endeavor is to help students in preparation of Banking exams like
SBI, IBPS, RBI, NICL, NIACL, LIC etc. Our endeavor is to provide all the relevant study materials, reference materials etc,
mainly free of cost or on little cost. We are committed to serve you better. Please visit our website & Facebook page daily to
get benefited. Facebook- www.facebook.com/ibpsprepadda

A) The car covers a distance 120 km in 4 hours.


B) The car covers a distance 210 km in 7 hours.
59. A certain amount was to be distributed among A, B and C in the ratio 2 : 3 : 4 respectively but was
erroneously distributed in the ratio 7 : 2 : 5 respectively. As a result of this, B got Rs. 40 less. What is the
amount?
(1) Rs. 210/- (2) Rs. 270/(3) Rs. 230/- (4) Rs. 280/(5) None of these
60. Rachita enters a shop to buy ice-creams, cookies and pastries. She has to buy atleast 9 units of each.
She buys more cookies than ice-creams and more pastries than cookies. She picks up a total of 32 items.
How many cookies does she buy?
(1) Either 12 or 13 (2) Either 11 or 12
(3) Either 10 or 11 (4) Either 9 or 11
(5) Either 9 or 10
61.. The fare of a bus is Rs. X for the first five kilometers and Rs. 13/-per kilometer thereafter. If a
passenger pays Rs. 2402/- for a journey of 187 kilometers, what is the value of X?
(1) Rs. 29/- (2) Rs. 39/- (3) Rs. 36/(4) Rs. 31/- (5) None of these
62.The ages of Anjali and Renu are in the ratio of 6 : 7 respectively. After 6 years, the ratio of their ages
will be 15 : 17. What is the age of Renu ? (In years)
1) 28 2) 24 3) 32 4) 42 5) None of these
63. Mohan borrowed a sum of ` 6300 from Vikas at the rate of 14% for 3 year. He then added some
more money to the borrowed sum and lent it to Mohit at the rate of 16% of simple interest for the same
time. If Mohan gained ` 618 in the whole transaction, then what sum did he lent to Mohit ?
1) ` 6500 2) ` 7200 3) ` 7800 4) ` 7000 5) None of these
64.A merchant buys tea at Rs. 126, Rs. 128 and Rs. 132 per Kg and mixes them in the proportion 6 : 3 : 1.
At what price must he sell the mixtures so as to make a profit of 25%?
14

Ibpsprepadda is an initiative by Bankers & students. Our endeavor is to help students in preparation of Banking exams like
SBI, IBPS, RBI, NICL, NIACL, LIC etc. Our endeavor is to provide all the relevant study materials, reference materials etc,
mainly free of cost or on little cost. We are committed to serve you better. Please visit our website & Facebook page daily to
get benefited. Facebook- www.facebook.com/ibpsprepadda

1) 159 2) 163 3) 149 4) 152 5) None of these


65. A dealer mixed two varieties of tea costing Rs. 121.25 per kg with tea costing Rs. 126.25 per kg. and
sells the mixture at Rs. 141.70 per kg. and earns a profit of 15% on his price. In what proportion does he
mix them?
1) 1:2 2) 3:2 3) 5:7 4 ) 2:3 5) None of these

Test III

Reasoning Ability
Directions (66 - 70): In these questions, certain symbols have been used to indicate relationships
between elements as follows:
'P Q' means 'P is neither greater than nor equal to Q.
'P $ Q' means 'P is neither smaller than nor equal to Q.
'P @ Q' means 'P is not smaller than Q.
'P % Q means P is not greater than Q.
'P Q' means 'P is neither greater than nor smaller than Q.
In each question, four statements showing relationships have been given, which are followed by three
conclusions I, II and III. Assuming that the given statements are true, find out which conclusion(s) is /are
definitely true?

66. Statement:
R % T, T K, K $ M, M @ V
Conclusions:

I. M T

II. R K

III. K $ V

1) None is true 2) Only I and II are true


3) Only II and III are true 4) Only I and III are true
15

Ibpsprepadda is an initiative by Bankers & students. Our endeavor is to help students in preparation of Banking exams like
SBI, IBPS, RBI, NICL, NIACL, LIC etc. Our endeavor is to provide all the relevant study materials, reference materials etc,
mainly free of cost or on little cost. We are committed to serve you better. Please visit our website & Facebook page daily to
get benefited. Facebook- www.facebook.com/ibpsprepadda

5) All I, II and III are true

67. Statements:
W $ N N R, R @ K, K % F
Conclusions:

I. F R

II. R $ W

III. N K

1) None is true 2) Only I is true 3) Only II is true


4) Only III is true 5) Only II and III are true

68. Statements:
F @ M, M % W, W $ R, R V
Conclusions:

I. V $ M

II. F $ W

III. R F

1) None is true 2) Only I is true 3) Only II is true


4) Only III is true 5) Only I and III are true

69. Statements:
B $ D, D @ M, M % K, K R
Conclusions: I. R $ M

II. K@D

III. B $ M

1) Only I and II are true 2) Only I and III are true


3) Only II and III are true
4) All I, II and III are true 5) None of these

70. Statements:
N@W, WK, KV, V$F
Conclusions:

I. KN

II. K@N

III. FW

1) Only I is true 2) Only II is true 3) Only III is true


4) Only either I or II is true 5) None of these

16

Ibpsprepadda is an initiative by Bankers & students. Our endeavor is to help students in preparation of Banking exams like
SBI, IBPS, RBI, NICL, NIACL, LIC etc. Our endeavor is to provide all the relevant study materials, reference materials etc,
mainly free of cost or on little cost. We are committed to serve you better. Please visit our website & Facebook page daily to
get benefited. Facebook- www.facebook.com/ibpsprepadda

Directions (Q. 71 75): Study the following information carefully and answer the questions that
follows :
P, Q, R, S, T, U, V and W are eight enclosures in the zoo. Eight animals viz. Lion, Panther, Bison, Bear,
Deer, Leopard, Tiger and Monkey are housed in each enclosure but not necessarily in the same order. All
the enclosures are around a circular chamber but the lock of each enclosure is in the outward direction
with respect to the circular chamber. Enclosures T and V are not in neighbor position. Bear is housed in
enclosure V. There are two enclosures between the enclosures in which Panther and Bison are housed.
The enclosure in which bison is enclosed placed second to the left of enclosure P. There are two
enclosures between V and the enclosure in which monkey is housed. W and S are immediate neighbors
of each other. The enclosure in which leopard is housed, is on the immediate right of the enclosure in
which tiger is enclosed. Leopard and deer are not housed in enclosure U. U is not an immediate
neighbor of the enclosure in which monkey is enclosed. The enclosure in which leopard is housed
cannot placed with the enclosure in which lion is housed.
Enclosure T is second to the left of the enclosure in which panther is housed. Neither monkey nor
panther is housed in R. S and Q are not at neighbor position. Q in which bison is not housed placed on
the immediate left of enclosure U. There is only one enclosure placed between Q and the enclosure in
which monkey is housed and that enclosure can never be P. There is only one enclosure placed between
those two enclosures in which tiger and monkey are housed.
71. Which of the following animal is housed in enclosure U ?
1) Bear 2) Lion 3) Bison 4) Leopard 5) Can't be determined
72.. Tiger is housed in which of the following enclosures ?
1) T 2) V 3) Q 4) S 5) None of these
73. Which of the following pairs of enclosures and animals is definitely true ?
1) V Tiger 2) R Leopard 3) Q Lion 4) W Panther 5) None of these
74.. How many enclosures are placed between S and W when counted in clockwise direction from W ?
1) Six 2) Three 3) Four 4) Five 5) None
75.. Which of the following is the position of enclosure P with respect to the enclosure in which deer is
housed ?
1) Third to the right 2) Fourth to the left 3) Third to the left 4) Second to the right 5) None of these

17

Ibpsprepadda is an initiative by Bankers & students. Our endeavor is to help students in preparation of Banking exams like
SBI, IBPS, RBI, NICL, NIACL, LIC etc. Our endeavor is to provide all the relevant study materials, reference materials etc,
mainly free of cost or on little cost. We are committed to serve you better. Please visit our website & Facebook page daily to
get benefited. Facebook- www.facebook.com/ibpsprepadda

Directions (Q. 7680): In each questions below are given three statements followed by two conclusions
numbered I and II. You have to take the given statements to be true even if they seem to be at variance
with commonly known facts. Read all the conclusion and then decide which of the given conclusions
logically follows from the given statements, disregarding commonly known facts. Give answer :
1) If only conclusion I follows. 2) If only conclusion II follows.
3) If either conclusion I or conclusion II follows. 4) If neither conclusion I nor conclusion II follows.
5) If both conclusion I and II follow.
(76 77) :
Statements : All rows are columns. Some columns are straight. Some straight are queue.
75. Conclusions: I. Some rows being queue is a possibility. II. All rows are straight.
76. Conclusions: I. Some columns can never be rows. II. At least some queues are columns.
(7879) :
Statements : No blue is sea. Every sea is river. Some rivers are fountains.
78. Conclusions: I. All blue are possibly fountain. II. Some sea are fountain.
79.. Conclusions: I. No river is sea. II. There is a possibility that some blue are not fountain.
80. Statements: All managers are professionals. All professionals are engineers. Some engineers are
good.
Conclusions: I. All engineers being manager is a possibility. II. Some professionals are not managers.
Directions for questions 81 to 85: Answer the questions on the basis of the information given below.
Five friends - Turner, Kapranos, Okereke, Casablancas and Banks - were discussing about experimental
rock bands of the 80s that have influenced and shaped their musical journey. The only bands involved in
their discussion were - Doy Jivision, Pothole Surfers, Reverb & The Dummymen, Smell Wasps and Fang
of Gore. Finally they concluded that each one of them dislikes exactly two bands, no two of them dislike
the same two bands and each band is liked by exactly three of them. Doy Jivision was disliked by
Okereke and Banks for being way too depressing. Kapranos, who is not exactly a fan of scholarly
disseminations on existential angst in music, disliked Pothole Surfers and Smell Wasps. There was
exactly one band that was liked by both Kapranos and Casablancas. Okereke, who hails from a
conservative middle class family, disliked Fang of Gore due to their association with radical post
structural Bolsheviks. There was exactly one band that was disliked by both Turner and Casablancas.
18

Ibpsprepadda is an initiative by Bankers & students. Our endeavor is to help students in preparation of Banking exams like
SBI, IBPS, RBI, NICL, NIACL, LIC etc. Our endeavor is to provide all the relevant study materials, reference materials etc,
mainly free of cost or on little cost. We are committed to serve you better. Please visit our website & Facebook page daily to
get benefited. Facebook- www.facebook.com/ibpsprepadda

81.Who among the following liked Pothole Surfers?


1.Turner
2.Kapranos
3. Banks
4. Cannot be determined
82.Which band was disliked by both Turner and Casablancas?
1.Pothole Surfers
2. Reverb & The Dummymen
3. Smell Wasps
4.Either Pothole Surfers or Fang of Gore
83.Banks disliked?
1. Doy Jivision and Pothole Surfers
2. Doy Jivision and Smell Wasps
3. Either (a) or (b)
4. None of these
Directions (Q. 8488): Study the following information carefully to answer the given questions :
In a certain code "environmental information is here" is written as 'tu fa lic li' 'here read given
important' is written as 'pi sa uic fa', 'awareness is necessarily important' is written as 'uic hi li no' and
'necessarily given environmental questions' is written as 'xo lic pi hi'.
84. What is the code for 'information' ?
1) tu 2) fa 3) lic 4) li 5) Can't be determined
85. Which of the following is the code for 'is necessarily read' ?
1) sa hi li 2) sa fa hi 3) no hi li 4) pi hi li 5) None of these
86. What does 'no' stand for ?
19

Ibpsprepadda is an initiative by Bankers & students. Our endeavor is to help students in preparation of Banking exams like
SBI, IBPS, RBI, NICL, NIACL, LIC etc. Our endeavor is to provide all the relevant study materials, reference materials etc,
mainly free of cost or on little cost. We are committed to serve you better. Please visit our website & Facebook page daily to
get benefited. Facebook- www.facebook.com/ibpsprepadda

1) is 2) awareness 3) necessarily 4) important 5) Can't be determined


87. Which of the following is represented by the code 'fa pi lic' ?
1) here is given 2) information here given 3) here given environmental
4) here read information 5) None of these
88. Which of the following may be the possible code for 'here exact awareness information' ?
1) uic no tu qa 2) no qa ja fa 3) no fa tu qa 4) no fa tu uic 5) None of these

89. Statements: The Reserve Bank of India has the power to issue the currency notes in India. There has
always been the dilemma among the people that since the Reserve Bank of India has the power to issue
the currency notes then it should print the excess notes and distribute it among the people which will
help in reducing the poverty in the country. What could be the main reason that the Reserve Bank of
India not printing the excess currency notes.
I) Excess currency notes will increase the inflation in the country.
II) The Reserve Bank of India is not allowed to print the currency notes.
III) Excess currency will devaluate the value of the Indian rupee in the international market.
IV) Excess supply of money will reduce the amount of commodity in the market.

1) Only I 2) Only I and II 3) Only I, II and III 4) Only I, III and IV 5) All I, II, III and IV

90. Statements: The Prime Lending Rate is the rate of interest at which the commercial banks gives loan
to the credit worthy customers or the prime customers. Which among the following is/are the effect of
the prime lending rates introduced by the banks?
I) The prime lending rate helps in maintaining the old customers with the bank.
II) Prime lending rate helps in reduction of the risk with the banks.
III) Prime lending rate helps in improving the ratings of the bank.
IV) Prime lending rate helps in maintaining the transparency in the market.
20

Ibpsprepadda is an initiative by Bankers & students. Our endeavor is to help students in preparation of Banking exams like
SBI, IBPS, RBI, NICL, NIACL, LIC etc. Our endeavor is to provide all the relevant study materials, reference materials etc,
mainly free of cost or on little cost. We are committed to serve you better. Please visit our website & Facebook page daily to
get benefited. Facebook- www.facebook.com/ibpsprepadda

Only I 2) Only I and II 3) Only II 4) Only III 5) Only I, II and III


91. Statements: Afzal Guru was hanged in the early hours of February, 9. The date of execution was
fixed only the previous day, but the authorities choose no quicker means of communication than speed
post to inform his family in sopore of the final decision. The idea seemed to be that no one should know,
react or protest. Afzal himself was given no chance to challenge the President's rejection, through it is
open to judicial review.
Which among the following assumptions is implicit in the following statements ?
1) The Government did not want the family of Afzal Guru to meet him one last time.
2) The Government of India was worried about the protest and violence in the Kashmir Valley.
3) The Government did not want to waste time on the terrorist like Afzal Guru.
4) The Constitution of India does not allow that.
5) None of these
92. Statements: The return to office of an elected Prime Minister Nawaz Sharif in Pakistan after the
decadeand-a-half after he was deposed by the military is an important development for India as well as
the international community, which have stakes in the progress of democracy in Pakistan. Which among
the following conclusion supports the statement ?
1) Consolidation of democracy in Pakistan offers hope of arresting its decline towards extremism and
state failure.
2) It will help in controlling the menace of terrorism.
3) It will help in strengthening pluralism.
4) It will help in changing the equation between the armed forces and the civilian authority and
stabilising the polity.
5) All of the above
93. Statements: The sex ratio of India in the census of India 2011 has reduced to 940 females / 1000
males. Despite constant efforts of the government of India and the non government organisations the
sex ratio is not improving and the conditions of the women is degrading day-by-day in India.
What course of action government should take in order to improve the sex ratio in India ?
A) The government should make strict laws on the sex determination test before the birth of the child.
21

Ibpsprepadda is an initiative by Bankers & students. Our endeavor is to help students in preparation of Banking exams like
SBI, IBPS, RBI, NICL, NIACL, LIC etc. Our endeavor is to provide all the relevant study materials, reference materials etc,
mainly free of cost or on little cost. We are committed to serve you better. Please visit our website & Facebook page daily to
get benefited. Facebook- www.facebook.com/ibpsprepadda

B) Female infanticide and feticide should be banned.


C) The government should work upon to increase the female education in India.
D) More women reservation should be provided in India in all the sectors.
1) Only A 2) Only A, B and C 3) Only B and D 4) Only C and D 5) All of these
Directions (Q.9 498): Study the following information carefully to answer the given question :
A, B, C, D, E, F, G and H are sitting around a circular table but not facing the centre. C is second to the
right of B. B is fourth to the left of E. F and A are not immediate neighbors of B. F is third to the left of A.
H is not an immediate neighbor of A. G is not on the immediate left of A.
94. Which of the following is true about D ?
1) D is second to the left of F. 2) D is on the immediate left of A.
3) D is opposite of C. 4) D is third to the left of H. 5) None of these
95. What is the position of A with respect to B ?
1) Immediate left 2) Fifth to the left 3) Second to the right 4) Third to the right 5) None of these
96. Who is third to the left of C ?
1) G 2) A 3) D 4) B 5) E
97. Which of the following pairs are neighbor of G ?
1) H, B 2) D, A 3) A, B 4) Can't be determined 5) None of these
98. Who sits on the immediate left of E ?
1) C 2) F 3) H 4) D 5) None of these
99. P moves 10 km towards the East, turns right walks 2 km, turns right and covers a distance of 4 km. P
again turns left. After covering a distance of 5 km, he again turns to right and covers a distance of 6 km.
In which direction is P with respect to the starting point ?
1) North 2) South 3) SouthEast 4) SouthWest 5) None of these
100. Four of the following five are alike in a certain way and so form a group. Which is the one that does
not belong to that group?
1) Praries 2) Pampas 3) Steppes 4) Savana 5) Dawns
22

Ibpsprepadda is an initiative by Bankers & students. Our endeavor is to help students in preparation of Banking exams like
SBI, IBPS, RBI, NICL, NIACL, LIC etc. Our endeavor is to provide all the relevant study materials, reference materials etc,
mainly free of cost or on little cost. We are committed to serve you better. Please visit our website & Facebook page daily to
get benefited. Facebook- www.facebook.com/ibpsprepadda

SBI PO PRELIMS- SAMPLE PAPER-1 ANSWERS


Answers:
1.a
2.c
3.c
4.c
5.d
6.a
7.c
8.b
9-2
10-3
11.1
12.2
13.1
14.4
15.1
16.1
17.5
18.2
19.1
20.2
21.3
22.4

www.ibpsprepadda.com

Ibpsprepadda is an initiative by Bankers & students. Our endeavor is to help students in preparation of Banking exams like
SBI, IBPS, RBI, NICL, NIACL, LIC etc. Our endeavor is to provide all the relevant study materials, reference materials etc,
mainly free of cost or on little cost. We are committed to serve you better. Please visit our website & Facebook page daily to
get benefited. Facebook- www.facebook.com/ibpsprepadda

23.1 24.5 25.4 26.2 27.4 28.1 29.5 30.3 31.2 32.3 33.4 34.4 35.5 36.3 37.1 38.3 39.3 40.5 41.1 42.3 43.2
44.1 45.2 46.5 47.1 48.4 49.2 50.1 51.1 52.4 53.2 54.4 55.5 56.2 57.1 58.4 59.1 60.3 61.3 62.1 63.5 64.1
65.2
66.4
67.3
68.1
69.2
70.1
71.2
72.5
73.4
74.1
75.2
76.1
77.4
78.1
79.2
80.1
81.1
82.2
83.3 84.1 85.1 86.2 87.3 88.3 89.4 90.2 91.2 92.5 93.2 94.2 95.5 96.1 97.3 98.2 99.2 100.4

www.ibpsprepadda.com

Books and Authors (Latest released along with some important ones)
------------------------------------------------------------------------------------1. Obsession in Death- J D Robb
2. Hostage - by Kristina Ohlsson
3. Virat Kohli : Reliable Rebel - by Ayaz Memon , C. Rajshekar Rao
4. Mastermind - by Maria Konnikova
5. Samrat: How the Shiv Sena Changed Mumbai Forever - By: Sujata Anandan
6. Dilip Kumar: The Substance and the Shadow An Autobiography - By: Dilip Kumar
7. The Seven Spiritual Laws Of Success : A Pocket Guide to Fulfilling Your Dreams - By: Deepak Chopra
8. Endangered - by C J Box
9. The Matchmaker - by Elin Hilderbrand
10. The Buried Giant - by Kazuo Ishiguro
11. The Mime Order (English) - by Samantha Shannon
12. At the Helm: A Memoir - by V. Krishnamurthy
13. Bird in a Banyan Tree: My Story - by Bina Ramani
14. Kaun Banega Crorepati by Siddhartha Basu
15. Diary Of A Young Girl - by Anne Frank
16. Killing Cupid - by Louise Voss , Mark Edwards
17. Motive: An Alex Delaware Novel - by Jonathan Kellerman
18. The Grand Design - by Stephen Hawking ,Leonard Mlodinow
19. A Brief History of Time - by Stephen Hawking
20. Byculla to Bangkok - by Hussain Zaidi
21. Marissa Mayer and the Fight to Save Yahoo - By: Nicholas Carlson
22. The Longest Afternoon: The 400 Men Who Decided the Battle of Waterloo By: Professor Brendan Simms (University of Cambridge)
23. This Divided Island: Stories from the Sri Lankan War - By: Samanth Subramanian
24. How to Fail at Almost Everything and Still Win Big: Kind of the Story of My Life By: Scott Adams
25. Editor Unplugged: Media, Magnates, Netas and Me - By: Vinod Mehta
26. SUNIL GAVASKAR : CRICKET'S LITTLE MASTER 01 Edition -- By: Debasish
Datta
27. A Biography Of Rahul Dravid - by Devendra Prabhu
28. World Cricket Records: 2013 - By: Chris Hawkes
29. The Theory of Everything: The Origin and Fate of the Universe - By: Stephen Hawking
30. My Brief History - by Stephen Hawking
31. The Theory of Everything: The Origin and Fate of the Universe - By: Stephen Hawking
32. Not Just an Accountant: The Diary of the Nations Conscience Keeper - By:Vinod Rai
33. Women of the World: The Rise of the Female Diplomat - By: Helen McCarthy
34. Getting India Back on Track: An Action Agenda for Reform - By: Bibek Debroy,Ashley
J. Tellis and Reece Trevor
35. A Dangerous Place LP: A Maisie Dobbs Novel - By: Jacqueline Winspear
36. Unreal Elections - By: Karthik Laxman , C.S. Krishna
37. Gene Cloning and DNA Analysis: An Introduction - By: T. A. Brown
38. Total Recall - by Arnold Schwarzenegger
39. The Toi Story - by Sangita P. Menon Malhan
40. It's Not About the Bike: My Journey Back to Life - By: Lance Armstrong
41. Profit from the Core: A Return to Growth in Turbulent Times - By: Chris Zook
42. Statistics for Six Sigma Green Belts with Minitab and JMP - By: David M. Levine
43. The Big 100: The 100 Business Tools You Need to Succeed - By: Jeremy Kourdi

www.BankExamsToday.com

SBI PO Prelim 2015


Model Paper
By Ramandeep Singh

Ramandeep Singh
4/14/2015

SBI PO Prelim 2015 Model Paper


Quantitative Aptitude

1. The average age of husband, wife and their child 3 years ago was 27 years and that of wife and
the child 5 years ago was 20 years. The present age of the husband is:
a) 35 years
b) 40 years
c) 50 years
d) Data inadequate
e) None of these
2. A rectangular courty 3.78 metres long and 5.25 metres wide is to be paved exactly with square
tiles, all of the same size. What is the largest size of the tile which could be used for the purpose?
a) 14 cms
b) 21 cms
c) 42 cms
d) Data inadequate
e) None of these
3. Murugan, Prasanna and Arun invested Rs.8000, Rs.4000 and Rs.8000 respectively in a business.
Arun left after six months. If after eight months, there was a gain of Rs.4005, then what will be
the share of Prasanna?
a) Rs.890
b) Rs.1335
c) Rs.1602
d) Rs.1780
e) None of these
4. In how many ways a committee, consisting of 5 men and 6 women can be formed from 8 men
and 10 women?
a) 266
b) 5040
c) 1176
d) 86400
e) None of these
5. In a lottery, there are 10 prizes and 25 blanks. A lottery is drawn at random. What is the
probability of getting a prize?
a) 1/10
b) 2/5
c) 2/7
d) 5/7
e) None of these
6. A man is 24 years older than his son. In two years, his age will be twice the age of his son. The
present age of the son is
a) 14 years
b) 18 years
c) 20 years
d) 22 years
e) None of these
7. A shopkeeper expects a gain of 22-1/2% on his cost price. If in a week, his sale was of Rs.392,
what was his profit?
a) Rs.18.20
b) Rs.70
c) Rs.72
d) Rs.88.25
e) None of these
8. The sum of n terms of the series 1 + (1 + 3) + (1 + 3 + 5) + . is :
a)
d) Data inadequate
9.

b)
e) None of these

. 081 .484 /.0064 6.25 is equal to


a) 0.9
b) 0.99
d) 99
e) None of these

c)

c) 9

10. A boy goes to his school from his house at a speed of 3 km./hr and return at a speed of 2 km./hr.
If he takes 5 hours in going and coming, the distance between his house and school is
a) 5 km
b) 5.5 km
c) 6 km
www.BankExamsToday.com

Page 2

SBI PO Prelim 2015 Model Paper

d) 6.5 km
e) None of these
11. A can do a certain work in the same time in which B and C together can do it. If A and B together
could do it in 10 days and C alone in 50 days, then B alone could do it in:
a) 15 days
b) 20 days
c) 25 days
d) 30 days
e) None of these
12. If the circumradius of an isoceless triangle ABC is equal to AB (= AC), then angle A is equal to
a)
b)
c)
d)

e) None of these

13. If 10, 12 and x are sides of an acute angled triangle, how many integer values of x are
possible?
a) 7
b) 12
c) 9
d) 13
e) 11
14. A man can row upstream at 7 kmph and downstream at 10 kmph. Find mans rate in still water
and the rate of current?
a) 6.5, 1.2 km/hr
b) 8.5, 1.5 km/hr
c) 1.5, 1.6 km/hr
d) 7.5, 1.8 km/hr
e) None of these
Study the following table and answer the questions based on it.
Expenditures of a Company (in Lakh Rupees) per Annum Over the given Years.
Item of Expenditure
Year
Salary
Fuel and Transport
Bonus
Interest on Loans

Taxes

1998

288

98

3.00

23.4

83

1999

342

112

2.52

32.5

108

2000

324

101

3.84

41.6

74

2001

336

133

3.68

36.4

88

2002

420

142

3.96

49.4

98

15. What is the average amount of interest per year which the company had to pay during this period?
a) Rs.32.43 lakhs
b) Rs.33.72 lakhs
c) Rs.34.18 lakhs
d) Rs.36.66 lakhs
e) None of these
16. The total amount of bonus paid by the company during the given period is approximately what
percent of the total amount of salary paid during this period?
a) 0.1%
b) 0.5%
c) 1%
d) 1.25%
e) None of these
17. Total expenditure on all these items in 1998 was approximately what percent of the total
expenditure in 2002?
a) 62%
b) 66%
c) 69%
d) 71%
e) None of these
18. The total expenditure of the company over these items during the year 2000 is?
www.BankExamsToday.com

Page 3

SBI PO Prelim 2015 Model Paper

a) Rs.544.44 lakhs
d) Rs.478.87 lakhs

b) Rs.501.11 lakhs
e) None of these

c) Rs.446.46 lakhs

19. The ratio between the total expenditure on taxes for all the years and the total expenditure on fuel
and transport for all the years respectively is approximately?
a) 4 : 7
b) 10 : 13
c) 15 : 18
d) 5 : 8
e) None of these
20. On 6th March 2005 Monday falls. What was the day of the week on 6th March 2004?
a) Sunday
b) Saturday
c) Tuesday
d) Wednesday
e) None of these
21. At what angle the hands of a clock are inclined at 15 minutes past 5?
a) 58
b) 64
c) 67
d) 72

e) None of these

22. Two pipes A and B can fill a tank in 20 and 30 minutes respectively. If both the pipes are used
together, then how long will it take to fill the tank ?
a) 12 min
b) 15 min
c) 25 min
d) 50 min
e) None of these
23. A lent Rs. 5000 to B for 2 years and Rs. 3000 to C for 4 years on simple interest at the same rate
of interest and received Rs. 2200 in all from both of them as interest. The rate of interest per
annum is:
a) 5%
b) 7%
c) 7 1/8%
d) 10%
e) None of these
24. An agent gets a commission of 2.5% on the sales of cloth. If on a certain day, he gets Rs.12.50 as
commission, the cloth sold through him on that day is worth
a) Rs.250
b) Rs.500
c) Rs.750
d) Rs.1250
e) None of these
25. The cost of carpeting a room 18 m long with a carpet 75 cm wide at Rs.4.50 per metre is Rs.810.
The breadth of the room is :
a) 7 m
b) 7.5 m
c) 8 m
d) 8.5 m
e) None of these
26. Which one of the following is the common factor of (47 + 43 ) and (47
a) (47 43)
b) (47 + 43)
c) (47 + 43 )
d) Data inadequate
e) None of these

+ 43 ) ?

27. A Students was asked to find the arithmetic mean of the numbers 3, 11, 7, 9, 15, 13, 8, 19, 17,
21, 14 and x. He found the mean to be 12. What should be the number in place of x?
a) 3
b) 7
c) 17
d) 31
e) None of these
28. Which of the following is a pair of co-primes?
a) (16, 62)
b) (18, 25)
d) (23, 92)
e) None of these
www.BankExamsToday.com

c) (21, 35)

Page 4

SBI PO Prelim 2015 Model Paper

29. A camel pursue an elephant and takes 5 leaps for every 7 leaps of the elephant, but 5 leaps of
elephant are equal to 3 leaps of camel. What is the ratio of speeds of camel and elephant?
a) 21 : 25
b) 24 : 23
c) 25 : 21
d) 23 : 24
e) None of these
30. A, B and C jointly thought of engaging themselves in a business venture. It was agreed that A
would invest Rs. 6500 for 6 months, B, Rs. 8400 for 5 months and C, Rs. 10,000 for 3 months. A
wants to be the working member for which, he was to receive 5% of the profits. The profit earned
was Rs. 7400. Calculate the share of B in the profit.
a) Rs. 1900
b) Rs. 2660
c) Rs. 2800
d) Rs. 2840
e) None of these

Directions (Q. 31-35) The bar graph given below shows the sales of books (in thousand number) from
six branches of a publishing company during two consecutive years 2000 and 2001.
Sales of Books (in thousand numbers) from Six Branches - B1, B2, B3, B4, B5 and B6 of a
publishing Company in 2000 and 2001.

31. What is the ratio of the total sales of branch B2 for both years to the total sales of branch B4 for
both years?
a) 2 : 3
b) 3 : 5
c) 4 : 5
d) 7 : 9
e) None of these
32. Total sales of branch B6 for both the years is what percent of the total sales of branches B3 for
both the years?
a) 68.54%
b) 71.11%
c) 73.17%
d) 75.55%
e) None of these
33. What percent of the average sales of branches B1, B2 and B3 in 2001 is the average sales of
branches B1, B3 and B6 in 2000?
a) 75%
b) 77.5%
c) 82.5%
www.BankExamsToday.com

Page 5

d) 87.5%

SBI PO Prelim 2015 Model Paper


e) None of these

34. What is the average sales of all the branches (in thousand numbers) for the year 2000?
a) 73
b) 80
c) 83
d) 88
e) None of these
35. Total sales of branches B1, B3 and B5 together for both the years (in thousand numbers) is?
a) 250
b) 310
c) 435
d) 560
e) None of these

Reasoning Ability
36. In a row of 40 children, R is 11th from the right and there are 15 children between R and M. What
is Ms position from the left and of the row?
a) 14th
b) 15th
c) 13th
d) Cant be determined
e) None of these
37. In a certain code language how many are there is written as ka na ta da and many are
welcome here is written as na pa ni ka. How is how written in that code language?
a) ta
b) da
c) ta or da
d) Data inadequate
e) None of these
38. If the positions of the1st and the 5th digits of the number 83591427 are interchanged, similarly the
positions of the 2nd and the 6th digits are interchanged and so on then which of the following will
be the 2nd digit from the right end after the rearrangement?
a) 5
b) 3
c) 9
d) 2
e) None of these
39. How many such pairs of letters are there in the words ADJUSTING each of which has as many
letters between them in the word as in the English alphabet?
a) None
b) One
c) Two
d) Three
e) More than three
40. How many meaningful English words can be formed with the letters LBAE using each letter only
once in each word?
a) None
b) One
c) Two
d) Three
e) More than three
41. In a certain code BUILDER is written as JVCKSFE. How is SEALING written in that code?
a) BTFKHOJ
b) JOHKBFT
c) TFBKHOJ
d) BFTKJOH
e) None of these
42. If R denotes , T denotes , M denotes + and W denotes , then
27
T
15
R
3
W
4
M
6=?
a) 7
b) 13
c) 23
d) 1
e) None of these

www.BankExamsToday.com

Page 6

SBI PO Prelim 2015 Model Paper

43. In a certain code WAVE is written as 5%3* and WINS is written as 59@. How is SANE
written in that code?
a) 9@*
b) *%@
c) @%*
d) %@*
e) None of these
44. Which of the following is the middle digit of the 3rd highest among the five three digit numbers
given below?
368
931
472
715
647
a) 6
b) 3
c) 7
d) 1
e) 4
45. Among P, Q, R, S and T each having a different height, Q is shorter than only T and S is shorter
than P and Q. Who among them is the shortest?
a) R
b) S
c) P
d) Data inadequate
e) None of these
Directions (Q. 46-50) Study the following arrangement carefully and answer the questions given below.
BM%R3J@KDF69W4*NEP2$AY5IQZ#7UG
46. Which of the following is the 6th to the left of the 20th from the left end of the above arrangement?
a) J
b) Q
c) W
d) E
e) None of these
47. How many such consonants are there in the above arrangement, each of which is immediately
preceded by a symbol and immediately followed by a number?
a) None
b) One
c) Two
d) Three
e) More than three
48. If all the symbols and all the vowels are dropped from the above arrangement, which of the
following will be the 12th from the right end?
a) 9
b) 6
c) P
d) Y
e) None of these
49. How many such numbers are there in the above arrangement, each of which is immediately
preceded by a letter but not immediately followed by a letter?
a) None
b) One
c) Two
d) Three
e) More than three
50. What should come in the place of question mark (?) in the following series based on the above
arrangement?
MRJ F9 *E2
?
a) Y5I
b) YIQ
c) A5Q
d) YIZ
e) None of these
Directions (Q. 51-55) In each of the questions below are given four statements followed by four
conclusions numbered I, II, III and IV. You have to take the given statements to be true even if they seem
to be at variance from commonly known facts. Read all the conclusions and then decide which of the
given conclusions logically follows from the given statements disregarding commonly known facts.
51. Statements:

Some pencils are windows.


All windows are roads.

www.BankExamsToday.com

Page 7

SBI PO Prelim 2015 Model Paper

Some roads are cups.


All cups are chains.
Conclusions:
I. Some chains are pencils.
II. Some cups are pencils.
III. Some chains are windows.
IV. Some roads are pencils.
a) None follows
b) Only II follows
c) Only IV follows
d) Only III and IV follow
e) Only III follows
52. Statements:

Some beds are mirrors.


Some mirrors are dolls.
Some dolls are cheques.
Some cheques are pins.
Conclusions:
I. Some pins are dolls.
II. Some cheques are beds.
III. Some cheques are mirrors.
IV. Some dolls are beds.
a) None follows
b) Only I follows
c) Only II follows
d) Only III follows
e) Only IV follows

53. Statements:

All chocolates are holders.


No holder is lamp.
Some lamps are desks.
All desks are pens.
Conclusions:
I. Some pens are holders.
II. Some desks are lamps.
III. No pen is holder.
IV. Some pens are chocolates.
a) Only I follows
b) Only II follows
c) Only III follows
d) Only either I or III follows
e) Only either I or III and II follow

54. Statements:

All glasses are rooms.


Some rooms are planes.
All planes are ducks.
Some ducks are lanterns.
Conclusions:
I. Some lanterns are planes.
II. Some ducks are rooms.
III. Some rooms are glasses.
IV. Some ducks are glasses.
a) Only I and II follow
b) Only II and III follow
c) Only I, II and III follow

www.BankExamsToday.com

Page 8

SBI PO Prelim 2015 Model Paper

d) All I, II, III and IV follow


e) None of these
55. Statement:

Some chairs are tents.


Some tents are jugs.
All jugs are glasses.
All glasses are pots.
Conclusions:
I. Some pots are tents.
II. Some pots are chairs.
III. Some glasses are chairs.
IV. Some glasses are tents.
a) Only I and II follow
b) Only II and III follow
c) Only I and III follow
d) Only I and IV follow
e) None of these

Directions (Q. 56-60) In each question below is given a group of letters followed by four combinations of
digits/symbols numbered (a), (b), (c) and (d). You have to find out which of the combinations correctly
represents the group of letters based on the following coding system and the conditions that follow and
mark the number of that combination as your answer. If none of combinations correctly represents the
group of letter, mark (e) i.e., None of these as your answer.
Letter
W
Digit/Symbol 5
code

P
6

J
9

Q
1

E
2

T
3

I
@

A
4

F
8

D
%

B
*

V
7

M
#

H
$

Conditions:
(i)
If the 1st letter is a consonant and the 4th letter is a vowel both are to be coded as the code for
the vowel.
(ii)
If the 2nd letter is a vowel and the last letter is a consonant both are to be coded as !.
(iii)
If both the 1st and the last letters are consonants both their codes are to be interchanged.
56. MBUVWE
a) #*#52
d) #!75!

b) 7*#52
e) None of these

c) #*752

57. AJBMFU
a) 49*48
d) 9*#84

b) #9*#8
e) None of these

c) 49*#8

58. AEIMVH
a) 42@#7$
d) 4!@#7!

b) 42@47$
e) None of these

c) #2@47$

59. THAFIQ
a) 3$48@3
d) 3$48@1

b) 1$48@3
e) None of these

c) 1$48@1

60. WMEIJU
a) @#2@9

b) 5#2@9

c) @#259

www.BankExamsToday.com

Page 9

d) 5#259

SBI PO Prelim 2015 Model Paper


e) None of these

Directions (Q. 61-65) A word and number arrangement machine, when given an input line of words and
numbers rearranges them following a particular rule in each step. The following is an illustration of an
input and rearrangement.
Input:
17 put show on 39 27 85 gold
Step I:
show 17 put on 39 27 85 gold
Step II:
show 85 17 put on 39 27 gold
Step III:
show 85 put 17 on 39 27 gold
Step IV:
show 85 put 39 17 on 27 gold
Step V:
show 85 put 39 on 17 27 gold
Step VI:
show 85 put 39 on 27 17 gold
Step VII:
show 85 put 39 on 27 gold 17
And step VII is the last step of the rearrangement of the above input.
As per the rules followed in the above steps, find out in each of the following questions the appropirate
step for the given input.
61. Input: glass full 15 37 water now 85 67
Which of the following will be Step VI of the above input?
a) water 85 now 67 full glass 15 37
b) water 85 now 67 glass full 15 37
c) water 85 now 67 glass full 15 37
d) There will be no such step
e) None of these
62. Step II of an input is: ultra 73 12 16 mail sort 39 kite
Which of the following steps will be the last but one?
a) VIII
b) IX
c) VII
d) VI
e) None of these
63. Step III of an input is: win 75 voice 15 39 store gap 26
Which of the following is definitely the input?
a) voice 15 win 75 39 store gap 26
b) voice win 75 15 39 store gap 26
c) 15 75 win voice store gap 26
d) Cant be determined
e) None of these
64. Step II of an input is: tube 83 49 34 garden flower rat 56
How many steps will be required to complete the rearrangement?
a) Four
b) Five
c) Six
d) Three
e) None of these
65. Input: hunt for 94 37 good 29 48 book
How many steps will be required to complete the rearrangement?
a) Four
b) Five
c) Six
d) Seven
e) None of these
Directions (Q. 66-70) Study the following information carefully and answer the question given below.
www.BankExamsToday.com

Page 10

SBI PO Prelim 2015 Model Paper

A, B, C, D, E, F, G and H are sitting around a circle facing the centre. B is 2nd to the right of D
who is 3rd to the right of F. C is 2nd to the left of A who is 2nd to the left of F. G is 3rd to the right of E.
66. In which of the following combinations is the 1st person sitting between the 2nd and the 3rd
persons?
a) GCD
b) FGH
c) EFH
d) ABE
e) None of these
67. Who is 3rd to the right of H?
a) G
d) Data inadequate

b) D
e) None of these

c) C

68. Who is to the immediate right of A?


a) B
b) E
d) Data inadequate
e) None of these

c) F

69. What is Hs position with respect to B?


a) 5th to the right
b) 3rd to the left
rd
d) 3 to the right
e) 4th to the left

c) 5th to the left

70. Who is to the immediate left of G?


a) H
b) F
d) Data inadequate
e) None of these

c) D

English Language
Directions (Q. 71-80) Read the following passage carefully and answer the questions given
below it.
A long time ago, on a big tree in the lap of the mountain, lived a bird named Sindhuka. It was a
rather special bird because its droppings turned into gold as soon as they hit the ground.
One day, a hunter came to the tree in search of prey and he saw Sindhuka's droppings hit the
ground and turn into gold. The hunter was struck with wonder. He though, "I have been hunting birds and
small animals since I was a boy, but in all my 80 years, I have never seen such a miraculous creature. He
decided that he had to catch the bird somehow. He climbed the tree and skillfully set a trap for the bird.
The bird, quite unaware of the danger it was in, stayed on the tree and sang merrily. But it was soon
caught in the hunter's trap. The hunter immediately seized it and shoved it into a cage.
The hunter took the bird home joyfully. But as he had time to think over his good fortune later, he
suddenly realised, "If the king comes to know of this wonder, he will certainly take away the bird from
me and he might even punish me for keeping such a rare treasure all to myself. So it would be safer and
more honourable if I were to go to the king and present the unique bird to him," The next day, the hunter
took the bird to the king and presented it to him in court with great reverence. The king was delighted t o
receive such an unusual and rare gift. He told his courtiers to keep the bird safe and feed it with the best
bird food available.
The king's prime minister though, was reluctant to accept the bird. He said "O Rajah, how can
you believe the word of a foolish hunter accept this bird? Has anyone in our kingdom ever seen abird
dropping gold? The hunter must be either crazy or telling lies. I think it is best that you release the bird
from the cage." After a little thought, the king felt that his prime minister's words were correct. So he
www.BankExamsToday.com

Page 11

SBI PO Prelim 2015 Model Paper

ordered the bird to be released. But as soon as the door of the cage was thrown open, the bird flew out,
perched itself on a nearby doorway and defecated. To everyone's surprise, the dropping immediately
turned into gold. The king mourned his loss.
71. Which of the following is possible the most appropriate title for the story?
a) The Skilled Hunter
b) The Kings Prime Minister
c) The Kings Defeat
d) The Bird with the Gold Dropping
e) The Trials and Tribulations of the Foolish Bird Sindhuka
72. Which of the following emotions made the hunter gift the bird to the king?
a) Respect
b) Joy
c) Pride
d) Fear
e) Awe
73. Which of the following is true according to the story?
a) Birds like Sindhuka were very common in the area near the mountain
b) Sindhuka remained caged for the rest of its life
c) Sindhuka was unaware of the trap laid by the hunter
d) The King, when told to not accept the bird, did not listen to his Prime Minister
e) All are true
74. Why was the kings Prime Minister reluctant to accept the bird?
a) He believed that the bird would die if caged
b) He know about the hunters habit of lying
c) He believed that the bird would bring bad luck to the king
d) His sources had informed him that the hunter was crazy
e) None of these
75. How did the hunter find Sindhuka?
a) He had read stories about the bird and had set traps at various locations in the city
b) He followed the birds droppings
c) He was on the lookout for a prey when he chanced upon it
d) People from the city had informed him about the birds whereabouts
e) He was attracted by the birds calls
Directions (Q. 76-78) Choose the word which is most similar in meaning to the word/group of words
printed in bold as used in the passage.
76. Rather
a) Regular
d) But

b) Quite
e) Known

c) Instead

77. Release
a) Free
d) Let expire

b) Vacate
e) Make public

c) Vent

78. Reverence
a) Respect
d) Hope

b) Detail
e) Remembrance

c) Astonishment

www.BankExamsToday.com

Page 12

SBI PO Prelim 2015 Model Paper

Directions (Q. 79-80) Choose the word which is most opposite in meaning to the word printed in bold as
used in the passage.
79. Reluctant
a) True
d) Hesitant

b) Clever
e) Keen

c) Averse

80. Skilfully
a) Angrily
d) Cheaply

b) Haphazardly
e) Deftly

c) Highly

Directions (Q. 81-85): Rearrange the following six sentences (A), (B), (C), (D), (E) and (F) in the
proper sequence to form a meaningful paragraph and then answer the questions given below.
A. The researchers in these companies claim that they could do better by allowing their employees to
doze off at work place.
B. The dreams, while at work, are thus helpful to solve crucial problems.
C. Would you believe that some UK based companies are arranging for bed at the work place?
D. The reason, they claim, could be that dreams produce creative solutions.
E. We only hope that these crucial problems in UK are different from those of ours.
F. But it is true and is considered as a step to improve quality of their products.
81. Which of the following should be the First sentence after rearrangement?
a) A
b) B
c) C
d) D
e) None of these
82. Which of the following should be the Third sentence after rearrangement?
a) A
b) B
c) C
d) D
e) None of these
83. Which of the following should be the Fourth sentence after rearrangement?
a) A
b) B
c) C
d) D
e) None of these
84. Which of the following should be the Fifth sentence after rearrangement?
a) A
b) B
c) C
d) D
e) None of these
85. Which of the following should be the Sixth sentence after rearrangement?
a) A
b) B
c) C
d) E
e) None of these
Directions (Q. 86-90) Read this sentence to find out whether there is any grammatical mistake/error
in it. The error, if any, will be in one part of the sentence. Mark the part with the error as your
answer. If there is no error, mark No error as your answer. (Ignore the errors of punctuation if
any).
86. Attributing rise in inflation partly for withholding of food stocks by traders/the minister said
that/he was committed/to easing this supply side bottleneck.
a) Attributing rise in inflation partly for withholding of food stocks by traders
b) The minister said that
www.BankExamsToday.com

Page 13

SBI PO Prelim 2015 Model Paper

c) He was committed
d) To easing this supply side bottleneck.
e) No error

87. Indias largest utility vehicle and tractor maker/is again in the race to acquire/for stake in Swedish
company/which is a premium car maker.
a) Indias largest utility vehicle and tractor maker
b) Is again in the race to acquire
c) For stake in Swedish company
d) Which a premium car maker
e) No error
88. With sale of branded or premium petrol becoming almost nil/due to high duties,/a government
appointed panel has recommended/slashing excise duty to make them at par with regular fuel.
a) With sale of branded or premium petrol becoming almost nil
b) Due to high duties
c) A government appointed panel has recommended
d) Slashing excise duty to make them at par with regular fuel
e) No error
89. Keeping in mind/that power cuts are on different days in different areas/the change in the factory
law would enable individual factories within an area/to determining their own weekly holidays.
a) Keeping in mind
b) That power cuts are on different days in different areas
c) The change in the factory law would enable individual factories within an area
d) To determining their own weekly holidays
e) No error
90. Police officers have refused on identify the bystander,/who is the only eyewitness to the
crime,/but have said that the investigating team would explore/if he could be a witness in the
case.
a) Police officers have refused on identify the bystander
b) Who is the only eyewitness to the crime
c) But have said that the investigating team would explore
d) If he could be a witness in the case
e) No error
Directions (Q. 91-95): Below the four words are given. One of these four words may be wrongly spelt.
Find out the word which is wrongly spelt, if there is any. The number of that word is your answer. If all
the words are correctly spelt mark All correct as the answer.
91. Below the four words are given. One of these four words may be wrongly spelt. Find out the
word which is wrongly spelt, if there is any. The number of that word is your answer. If all the
words are correctly spelt mark All correct as the answer.
a) Adventure
b) Demonstration
c) Environment
d) Innosent
e) All Correct
92. Below the four words are given. One of these four words may be wrongly spelt. Find out the
word which is wrongly spelt, if there is any. The number of that word is your answer. If all the
words are correctly spelt mark All correct as the answer.
a) Limitasion
b) Dependable
c) Miniature
www.BankExamsToday.com

Page 14

d) Qualitative

SBI PO Prelim 2015 Model Paper


e) All Correct

93. Below the four words are given. One of these four words may be wrongly spelt. Find out the
word which is wrongly spelt, if there is any. The number of that word is your answer. If all the
words are correctly spelt mark All correct as the answer.
a) Lucrative
b) Ancestral
c) Performanse
d) Incidentally
e) All Correct
94. Below the four words are given. One of these four words may be wrongly spelt. Find out the
word which is wrongly spelt, if there is any. The number of that word is your answer. If all the
words are correctly spelt mark All correct as the answer.
a) Futility
b) Separasion
c) Embarrassment
d) Positively
e) All Correct
95. Below the four words are given. One of these four words may be wrongly spelt. Find out the
word which is wrongly spelt, if there is any. The number of that word is your answer. If all the
words are correctly spelt mark All correct as the answer.
a) Tournament
b) Enhancement
c) Amazingly
d) Continuation
e) All Correct
Directions (Q. 96-100): Rearrange the following six sentences (A), (B), (C), (D) and (E) in the
proper sequence to form a meaningful paragraph and then answer the questions given below.
A. Therefore, it is important to source a large part of economic growth in agriculture, in rural nonagricultural activities and in productive expansion of the informal sector which all have high
employment elasticities, as well as in an export strategy based on labour intensive exports.
B. It is important because it creates more resources and has the potential of creating more space for
the involvement of the poor.
C. If the growth is sourced upon those sectors of the economy or those activities that have a natural
tendency to involve the poor in their expansion, such growth helps poverty eradication.
D. Economic growth is important.
E. But this involvement depends on the sources of growth and the nature of growth.
96. Which of the following should be the First sentence after rearrangement?
a) A
b) B
c) C
d) D
e) E
97. Which of the following should be the Second sentence after rearrangement?
a) E
b) D
c) C
d) B
e) A
98. Which of the following should be the Third sentence after rearrangement?
a) A
b) B
c) C
d) D
e) E
99. Which of the following should be the Fourth sentence after rearrangement?
a) E
b) D
c) C
d) B
e) A
100.
a) A

Which of the following should be the Fifth sentence after rearrangement?


b) B
c) C

www.BankExamsToday.com

Page 15

SBI PO Prelim 2015 Model Paper

d) D

e) E

Solutions:
1. Sum of the present ages of husband, wife and child = (27 x 3 + 3 x 3) years = 90 years.
Sum of the present ages of wife and child = (20 x 2 + 5 x 2) years = 50 years.
Husband's present age = (90 - 50) years = 40 years.
2. Largest size of the tile.
HCF of 378 cm and 525 cm = 21 cms.
3. Murugan : Prasanna : Arun

= (8000 6) : (4000 8) : (8000 8)


= 48 : 32 : 64
=3:2:4
= Rs.4005
= Rs.890

Kamals share

= (8C5 x 10C6)
= (8C3 x 10C4)

=[

= 11760

4. Required number of ways

5. P (getting a prize) =

= Rs.[

6. Let the sons present age be x years.


Then, mans present age

7. C.P.

= (x + 24) years
= (x + 24) + 2 = x (x + 2)
= x + 26 = 2x + 4
= 22 years

392]

= Rs.[
392]
= Rs.320
Therefore, profit
= Rs.(392 320)
= Rs.72
8. 1 + 4 + 9 + 16 + . +
=

1 + 2 + 3 + 4 + . +

9. Sum of decimal places in the numerator and denominator under the radical sign being the same,
we remove the decimal.
Given exp.
= 81 484 / 64 625
=9
25
= 0.99

www.BankExamsToday.com

Page 16

10. Average speed

SBI PO Prelim 2015 Model Paper


= [2 3

+ 2) km./hr.

km./hr.

Distance travelled

=[
5] km.
= 12 km.
Distance between house and school = [ ] km
= 6 km.
11. (A + B)s 1 days work

Cs 1 days work

(A + B + C)s 1 days work


= [ + ] = = .. (i)
As 1 days work
= (B + C)s 1 days work .. (ii)
From (i) and (ii), we get 2 (As 1 days work) =
As days work

Bs 1 days work
=[ ]= =
So, B alone could do the work in 25 days.
12. Sin B =
=
=

[Given AB = AC = R]

=
B=

or

But, when B =
,C=
B+C>
So, B =
not possible

B=
C=

[AB = AC B = C]

[AB = AC B = C]

A = [

A=

13. For any triangle sum of any two sides must be greater than the third side.
The sides are 10, 12 and x.
From Rule 2, x can take the following values : 3, 4, 5, 6, 7, 8, 9, 10, 11, 12, 13, 14, 15, 16, 17, 18,
19, 20, 21 A total of 19 values.
When x = 3 or x = 4 or x = 5 or x = 6, the triangle is an OBTUSE angled triangle.
The smallest value of x that satisfies both conditions is 7. (10 + 7 > 12 )
The highest value of x that satisfies both conditions is 15. (10 + 12 + 15 )
When x = 16 or x = 17 or x = 18 or x = 19 or x = 20 or x = 21, the triangle is an OBTUSE angled
triangle.
Hence, the values of x that satisfy both the rules are x = 7, 8, 9, 10, 11, 12, 13, 14, 15. A total of 9
values.
www.BankExamsToday.com

Page 17

14. Rate in still water

SBI PO Prelim 2015 Model Paper


= (10 + 7) km./hr.
= 8.5 km./hr.
= (10 - 7) km./hr.
= 1.5 km./hr.

Rate of current

15. Average amount of interest paid by the company during the given period
.
.
.
.
.
= Rs. [
] lakhs
.

= Rs. [
] lakhs
= Rs.36.66 lakhs

16. Required percentage = [


=[
100] %
= 1%

17. Required percentage = [


.
.

=[
100] %
= 69.45

100] %

100] %

18. Total expenditure of company during 2000 = Rs.324 + 101 + 3.84 + 41.6 + 74) lakhs
= Rs.544.44 lakhs
19. Required ratio =
=
=
=

20. The year 2004 is a leap year. So, it has 2 odd days.
So, the day on 6th March 2005 will be 2 days beyond the day on 6th March 2004.
But 6th March 2005 is Monday
So, 6th March 2004 is Saturday.
21. Angle traced by hour hand in

hours = [

Angle traced by minute hand in 15 min. = [


So, required angle = [157
22. Part filled by A in 1 min.

] = 157

15] = 90

] 90 = 67

Part filled by B in 1 min.

Part filled by (A + B) in 1 min.

=[

=
Both the pipes can fill the tank in 12 minutes.

www.BankExamsToday.com

Page 18

SBI PO Prelim 2015 Model Paper

23. Let the rate be R% p.a.


Then, [

]+[

] = 2200

100R + 120R = 2200


R=[
] = 10
So, rate = 10%
24. Let the total sale be Rs. x
Then, 2.5% of x = 12.50
[

x] =
x=[

] = 500

25. Length of the carpet = [

]=[

] m = 180 m.

Area of the room = Area of the carpet = [180


So, breadth of the room = [

]=[

= 135

] m = 7.5 m

26. When n is odd, ( + ) is always divisible by (x + a)


So, each one of 47 + 43 and 47 + 43 is divisible by 47 + 43
27. Clearly, we have (3+11+7+9+15+13+8+19+17+21+14+x/12)
Number in place x is
137 + x = 144
x = 144 137
x=7
28. HCF of 18 and 25 is 1. So, they are co-primes.
29. Ratio of speed of camel and elephant = : = 15 :
= 25 : 21

15

30. For managing, A received = 5% of Rs. 7400 = Rs. 370.


Balance = Rs. (7400 - 370) = Rs. 7030.
Ratio of their investments = (6500 x 6) : (8400 x 5) : (10000 x 3)
= 39000 : 42000 : 30000
= 13 : 14 : 10
Bs share = Rs. [7030
] = Rs.2660
31. Required ratio =
32. Required percentage

=
=[

=
100] %
=[
100] %
= 73.17%

33. Average sales (in thousand number) of branches B1, B3 and B6 in 2000
= 80 + 95 + 70 =
www.BankExamsToday.com

Page 19

SBI PO Prelim 2015 Model Paper

Average sales (in thousand number) of branches B1, B2 and B3 in 2001


= 105 + 65 + 110 =
required percentage = [

/
/

100]% = [

100]% = 87.5%

34. Average sales of all the six branches (in thousand numbers) for the year 2000
= 80 + 75 + 95 + 85 + 75 + 70
= 80
35. Total sales of branches B1, B3 and B5 for both the years (in thousand numbers)
= 80 + 105 + 95 + 110 + 75 + 95 = 56
36. Option A
37. Option C
How many are there ka na ta da
Many are welcome here na pi ni ka (ii)
From equations (i) and (ii), many are na ka
how ta or da

(i)

38. Option A
Given number = 8
3
5
9
1
According to question, after rearrangement, new number =
1
4
2
7
8
3
5
9
2nd digit from right = 5
39. Option D
A
D
J
U
So, the pairs are AI and GI

40. Option B
41. Option E
42. Option B
Given arrangement = 27 T 15 R 3 W 4 M 6
According to question, letters converted into mathematical symbols
= 27 15 3 4 + 6 = 27 5 4 + 6
= 27 20 + 6 = 33 20 = 13
43. Option D
W
A
5
%
Similarly,
S
A

V
3

E
*

N
@

E
*

and

W
5

I
9

N
@

44. Option E
3rd highest number = 647
Middle digit = 4
45. Option D
According to height T > (P, Q) > (S, R)
www.BankExamsToday.com

Page 20

SBI PO Prelim 2015 Model Paper

So from question, it is not clear that which one is shorter S or R. So the given data is insuficient.
46. Option C
20 6 = 14th from left = W
47. Option B
Symbol Consonant
%
R

Number
3

48. Option D
After eliminating all vowels and symbols the arrangement will be
BMR3JKDF69W4NP2Y5QZ7G 12th from right end = y
49. Option D
Letters Number
F 6 9, W 4 *, P 2 $

Number/Symbol

50. Option D
51. Option C

Conclusions:

I. False
II. False
III. False
IV. True

Only IV follows
52. Option A

www.BankExamsToday.com

Page 21

SBI PO Prelim 2015 Model Paper

Conclusions:

I. False
II. False
III. False
IV. False

None follows
53. Option E

54. Option B

www.BankExamsToday.com

Page 22

SBI PO Prelim 2015 Model Paper

Conclusions:

I. False
II. True
III. True
IV. False

Only II and III follow


55. Option D

Conclusions:

I. True
II. False
III. False
IV. True

Only I and IV follows


56. Option E
M
B

www.BankExamsToday.com

E
Page 23

SBI PO Prelim 2015 Model Paper

57. Option C
A
J
4
9

B
*

M
#

F
8

58. Option D
According to condition (ii)
A
E
I
M
4
!
@
#

V
7

H
!

59. Option D
According to condition (iii)
T
H
A
F
1
$
4
8

I
@

Q
3

60. Option A
According to condition (i)
W
M
E
I
@
#
2
@

J
9

61. Option D
Input:
glass full 15 37 water now 85 67
Step I:
water glass full 15 37 now 85 67
Step II:
water 85 glass full 15 37 now 67
Step III:
water 85 now glas full 15 37 67
Step IV:
water 85 now 67 glass full 15 37
Step V:
water 85 now 67 glass 37 full 15
Step V is the last step and step VI is not possible.
62. Option D
Step II:
ultra 73 12 16 mail sort 39 kite
Step III:
ultra 73 sort 12 16 mail 39 kite
Step IV:
ultra 73 sort 39 12 16 mail kite
Step V:
ultra 73 sot 39 mail 12 16 kite
Step VI:
ultra 73 sort 39 mail 16 12 kite
Step VII:
ultra 73 sort 39 mail 16 kite 12
So last step is VII and last but one step is step VI.
63. Option D
64. Option A
Step II:
tube 83 49 34 garden flower rat 56
Step III:
tube 83 rat 49 34 garden flower 56
Step IV:
tube 83 rat 56 49 34 garden flower
Step V:
tube 83 rate 56 garden 49 34 flower
Hence four steps will be required to complete the rearrangement.
65. Option B
Input:
Step I:
Step II:

hunt for 94 37 good 29 48 book


hunt 94 for 37 good 29 48 book
hunt 94 good for 37 29 48 book

www.BankExamsToday.com

Page 24

SBI PO Prelim 2015 Model Paper

Step III:
hunt 94 good 48 for 37 29 book
Step IV:
hunt 94 good 48 for 37 29 book
Step V:
hunt 94 good 48 for 37 book 29
Hence five steps will be required to complete the arrangement.
66. Option D
67. Option C
68. Option B
69. Option E
70. Option A
71. Option D
72. Option D
73. Option C
74. Option E
75. Option C
76. Option B
77. Option A
78. Option A
79. Option E
80. Option E
81. Option C
82. Option A
83. Option D
84. Option B
85. Option D
86. Option A
87. Option B
88. Option A
89. Option E
90. Option A
91. Option D
92. Option A
93. Option C
94. Option B
95. Option E
96. Option D
97. Option C
98. Option E
99. Option D
100.
Option A

www.BankExamsToday.com

Page 25

Advanced Reasoning question


P,Q,R,S,T,V,and W are Sitting in a straight line facing north. each one of them lives on a different
floor in the same building which is numbered from one to seven .Q sits fourth to the left of the person
living on the 6th floor. either Q or the person living on the 6th floor sits at the extreme ends of the
line.
Only one person sits between Q and W.W lives on the 3rd floor. the person living on 1st floor sits
third to right of S.S is not an immediate neighbour of W. Only one person sits between T and the
person who lives on the 2nd floor.
P and R are immediate neighbours of each other. P does not live on the 6th floor. one who lives on
the 5th floor sits third to right of the one who lives on the 7th floor.
Questions
36. Who amongst the following lives on the 4th floor?
a)P
b)Q
c)R
d)S
37. On which of the following floors does T lives?
a)1st
b)2nd
c)5th
d)6th
38. How many floors are there between the floor on which V and P lives?
a)one
b)two
c) three
d)four

ANSWERS
36 TO 38: PERSON AND FLOOR
S 4- V 2 - Q 7- T -1 W-3 P-5 R-6 (SVQTWPR)
36. D (S) 37. 1 (T) 38. C (three floors)

ANALYTICAL DECISION MAKING


Analytical Decision Making is based on a set of relationships laid out, generally arbitrarily,
from which new information can be deduced. This involves two steps-first of analysis and
second of reasoning. Analytical decision making deals with questions in which you have to
decide upon the course of action taken upon a candidate who has
applied for a post or membership to an institution keeping in mind the essential requisites and the
data given for the candidate. We can classify such questions into a few major categories.
Category I
In this type a vacancy is being declared. The necessary qualifications required by the
recruiting agencies are given with certain exceptions. The qualifications and the merits of

the candidates are mentioned. The decision about each candidate has to be made from amongst
the five choices given, which state the courses of action to be taken as per the candidates
potential.
Category II
Here the eligibility conditions for joining a course or availing certain benefits etc are given as
against the vacancies mentioned in the former category. The qualifications of the candidates are
also mentioned. The decision about each candidate is to be made from amongst the five answer
choices given.
EXERCISE
Qs. 1-10. Following are the criteria for selection of officers in an organisation.
The candidate must:
(i) have passed HSC examination in first class with at least 60% marks.
(ii) have passed graduation degree in any discipline with at least 55% marks.
(iii) have completed a certificate/diploma/degree course in Computer Science.
(iv) be not less than 21 years and not more than 30 years of age as on 1.7.2008.
If a candidate satisfies all the above-mentioned criteria except:
(a) at (ii) above but is a post-graduate, case may be referred to the Executive Director (ED).
(b) at (iii) above but has studied Computer Science as one of subjects of curriculum, case may be
referred to the Vice President (VP).
In each of the questions below, information of one candidate is given. You have to take one of
the following five decisions based on the information provided and the criteria and conditions
given above. You are not to assume anything other than the information provided in each
question. All these cases are given to you as on 1.7.2008. You have to indicate your decision by
marking answers to each question as follows:
Mark answer
(1) if the case is to be referred to Executive Head.
(2) if the case is to be referred to Vice President.
(3) if the candidate is to be selected.
(4) if the information is inadequate to take a decision.
(5) if the candidate is not to be selected.
1. Suresh, is a Commerce graduate passed with 57% marks. He had secured 73% marks in HSC.
He has studied Computer Science as one of the subjects at HSC. His date of birth is 22.9.1982.
2. Archana has passed BMS degree examination in second class with 58% marks and HSC in
first class with 65% marks. She has completed a diploma in Computer Science. She has
completed 35 years of age in November 2007.
3. Raj Grover has passed HSC exam in first class with 49% marks. Thereafter he did a 6 months
certificate course in Computer Science and presently is pursuing final year of engineering degree
examination. His date of birth is 28.12.1980.

4. Shamika Gupta is a Science graduate passed in 2006 with 47% at the age of 22 years. She had
scored 64% marks in HSC. She has also passed M.Sc. with 58% marks. She has done a
certificate course in computers.
5. Jasmine is a postgraduate in Computer Science passed in first class with 62% marks. She had
scored 81% marks in HSC. Her date of birth is 17.6.1979.
6. Vaishnavi is a B.A. passed in first class with 63% marks. She had passed HSC examination in
first class with 69% marks. She has also completed a certificate course in Computer Science with
a A grade. Her date of birth is 23.9.1984.
7. Anubhav Gokhale, is a B.Sc. with Computer Science passed in second class with 58% marks.
He had passed HSC in first class with 76% marks. He has completed 25 years of age in
December 2007.
8. Manish Chaudhary passed HSC examination in first class with 83% marks and B.Com. in
second class with 57% marks. He has joined a computer certificate course very recently. His
date of birth is 26.4.1982.
9. Uvaraj passed HSC examination in 2003 with 85% marks and B.Sc. Degree examination in
2006 with 69% marks. He has completed a certificate course in computer science with good
marks. His date of birth is 17.9.84.
10. Vandana Chowdhury is B.Com graduate passed in second class with 56% marks. She had
passed HSC in second class with 69% marks. She has also completed a Computer diploma with
56% marks. Her date of birth is 11.5.1982.
PRACTICE EXERCISEANALYTICAL DECISION MAKING
Qs. 1-6. Study the following information carefully and answer the questions given below:
Following are the conditions for admitting students for a Doctoral Programme in Management:
The applicant must
(i) be a post graduate in any discipline with at least 50 per cent marks.
(ii) have passed the eligibility test with at least 60 per cent marks.
(iii) have work experience of at least two years after completing post graduation.
(iv) not be less than twenty five years as on 1.12.2002.
In the case of an applicant who fulfils all other conditions except:
(a) at (i) above, but have completed M. Phil. program, his/her case is to be referred to Director.
(b) at (ii) above, but has work experience of more than five years, his/her case is to be referred to
Registrar.
In each of the following questions, detailed information of one applicant is provided. You have
to decide the correct
course of action based on the above conditions and the information provided in each question.
You are not to assume anything other than the information provided in each question. All these
cases are given to you as on
1.12.2002.

Mark answer:
(1) if the applicant is to be admitted.
(2) if the applicant is not to be admitted.
(3) if the case is to be referred to Director.
(4) if the case is to be referred to Registrar.
(5) if the information provided is inadequate to take a decision.
1. Barkha Jaiswal has secured 75 per cent marks in post graduation and has been working for
seven years after post graduation. She was born on 11th February,1974.
2. Ashok Upadhyaya has been working for three years after completing M. Phil. Programme. He
was born on 1st November, 1973. He has secured 60 per cent marks in the eligibility test and 45
per cent marks in post
graduation.
3. Shikha Trivedi was born on 19th April, 1976. She has passed post graduation and eligibility
test each with 55 per cent marks. She has been working for three years.
4. Neela Goel has completed her M. Phil. Programme after completing her post graduation with
65 per cent marks. She has secured 50 per cent marks in the eligibility test. She has been working
for six years after post graduation. She was born on 20th August, 1972.
5. Manoj Khanna has secured 50 per cent and 60 per cent marks in post graduation and eligibility
test respectively. He has been working for four years after his education and he was born on 11th
February, 1974.
6. Tarun Nagpal was born on 5th March, 1975. He has secured 85 per cent marks in the
eligibility test and 55 per cent marks in post graduation. He has been working for three years.
Directions. Q. 7-10: Read the following situations and choose the best possible alternative.
7.
You, a recruitment manager, are interviewing Mayank, a hard working young man, who
has problems in speaking fluent English. He has studied in vernacular medium schools and
colleges. Amongst the following options, what would you choose to do, if your company has
vacancies?
1) I would hire him at all costs.
2) I would hire him for a production or finance job but not for a marketing job, which
requires good communication skills.
3) I would ask him to improve his communication skills and come back again.
4) I would not hire him as he might be a burden on the orgainsation because of his
orgainsation because of his poor communication skills.
5) I would hire him for the job he is good at, and provide training in other areas.
8.
The city of Nagar has a population of 10 million 2 million amongst whom were rich, 3,
million poor and 5 million belonged to the middle class. Saundarya Cosmetics manufactured and
sold beauty product to the rich class at a premium price. Its products were very popular with

customers. Many people from middle and poor segments of population aspired to buy these
products but could not afford them because of high prices. Of late, sales growth was stagnating
in the following is the best option for Saundarya Cosmetics to maximize long-term profits?
1) Sell the same products at lower prices to middle and poor class.
2) Sell its products under different brand names to middle and poor class.
3) Sell similar products, of different quality standards with different brand names, to
middle class and poor class.
4) Continue to target rich only and hope that todays middle class would be tomorrows
rice class.
5) Target middle class as it is the largest segment and forget about the rich.
9.
Seema was a Finance Manager in an MNC and felt that gender discrimination at the
workplace hampered her career growth. Frustrated, she quit the job and started a company.
While starting her company, Seema decided that she would have an equal proportion of males
and females. Over the last six years, Seema emerged as a very successful entrepreneur and
expanded her business to eight locations in the country. However, Seema recently stated facing
an ethical dilemma because she realised that female employees were not willing to travel across
cities and work late hours, as the work required them to do so. Male employees did not hesitate
undertaking such work. Seema started to feel the pressure of reducing the proportion of female
employees. On the other hand, she is aware that equal representation was one of the strongest
reasons for her to have founded the company. What should she do as a conscientious female
entrepreneur?
1) See if unwilling female employees could be given assignments which do not require
travel and involve less overtime.
2) Reduce the number of female employees, as it is a business requirement. She should
not let anything after her business.
3) Let the status quo continue
4) Henceforth hire only male employees
5) She should close the business
10.
A database software manufacturing company found out that a product it has launched
recently had a few bugs. The product has already been bought by more than a million customers.
The company realized that bugs could cost its customer significantly. However, if it informs the
customers about the bug, if feared losing credibility. What would be the most ethical option for
the company?
1) Apologise and fix up the bug for all customer even if it has to incur losses.
2) Do not tell customers about bugs and remove them only when customers face
problems, even if it meant losses for the customers.
3) Keep silent and do nothing.
4) Keep silent but introduce an improved product that is bug free at the earliest.
5) Take the product off the marked and apologise to customers

Directions (Qs. 11-20): The following criteria have been drawn to finalise the list of candidates
to be called for an interview after the round of written examination for bank P.O., held by the
XYZ Bank of India. The candidate must:

1. hold a degree in any stream, from a recognised university, with a first class in graduation and
at least Second division in post-graduation.
2. be at least 21 years, as on 01.01.02.
3. possess at least 80% marks in the written exam of bank P.O.
4. have a computer course certificate for 1 year course or more.
5. be drawing at least Rs 4500 p.m. in his/her previous job.
6. be of Indian nationality.
In case of candidates qualifying all conditions except:
7. (2) above, the case will be referred to the General Manager
(GM).
8. (4) above, the case will be referred to the Director.
Based on these conditions, decide the course of action and give answers as:
(a) if candidate can be admitted for interview.
(b) if candidate cannot be admitted for interview.
(c) if candidate is to be referred to G.M.
(d) if candidate is to be referred to Director.
(e) if data are inadequate to decide the course of action.

11. Mr Rajneesh has been working in a computer company as a trainee for the past 2 years. He
secured 80% marks in graduation and 57% in post-graduation. His marks in written exam are
82% and his date of birth is 19.07.79. He was drawing
Rs 6000 p.m. salary. He is an Indian.

12. Meghna was born on January 5, 1983. She is an arts graduate with 66% marks in graduation
and II class in post-graduation. She is currently working with the company Arti and Sons Ltd,
since past 2 years and possesses 2 years computer certificate also. She is an Indian and obtained
87% marks in the P.O. exam.

13. Mr Rajiv has done B.Com with 62% marks and post-graduation with 58%. He is an NRI and
has been employed for the past 3 years, drawing a salary of Rs 10,000 p.m. He secured 70% in
the written P.O. exam. He was born in Nov 1978. He has the requisite computer certificate.

14. Sapna has done her graduation at the age of 20. She possesses the requisite percentage of
marks at graduation, post-graduation and P.O. exams. Currently she is drawing Rs 12,500 salary
per month. She obtained a 2-year computer degree in 2002. She is an Indian.

15. Aamir is a fresh post-graduate Indian, employed with the MNC "Crookes & Co", getting Rs
22,000 p.m. as salary. He obtained his computer degree in 2002, for a 4-year course. He was
born on 02.06.1980. His marks in graduation were 54% and in post-graduation 67%. He got 80%
in P.O. exam.

16. Champa is a science graduate, who got 88% marks in the P.O. exam. She topped in her
graduation and post-graduation exams held by the RST University. She was born on 16.07.1976.
She is working with the ABC Co for past 5 years, drawing currently Rs 27,000 p.m. She is an
Indian by birth.

17. Ritu, an Indian, topped the P.O. exam with 94% marks. She has done her B.Com and M.Com
from Panjab University with 86% and 74% marks. She is nearly 26 years old and possesses 5year computer degree. She is employed at Rajapura Agro Tech Ltd for the past four years and
gets a salary of Rs 10,000 per month at present. She was born on 11.12.1974.

18. Charu has done her M.Sc. with IInd class and graduated with 72% marks. She is working for
past 1 years. She also obtained a 1-year computer diploma certificate. She secured 81% marks in
the P.O. written exam and is currently running her 22nd year. She is an Indian national. At
present she is drawing Rs 5500 basic salary. She is very charming girl and knows about office
procedures.

19. Prasoon is living in India for the past 20 years. He has secured the required marks at
graduation as well as at post-graduation. He is a management trainee in a company which gives
him Rs 5000 p.m. as basic salary. He is an Indian by birth. He obtained 84% marks in the P.O.
Exam. He also holds the requisite computer proficiency-level certificate.

20. Mr Mohanlal has done his M.A. with IInd class and graduation in Arts with 76% marks. He
is working in a company and draws Rs 5000 basic salary per month. He also did a 6 months
computer course. He has secured 83% marks in the bank P.O. Exam. He is an Indian.
EXAMPLES
Directions (Q. 1-10): Study the following information carefully and answer the questions
based on it.
Course X is one of the favourite courses of the youth started by a university ABC. The duration
of the course is three years. Each years assignments consists of four papers of 100 marks each.
(i) To be declared pass it is necessary for a candidate to obtain 50% marks in each paper besides
fetching 55% marks of total marks as a grand total (sum of the marks of twelve papers).
(ii) If a candidate obtains 80% marks of total marks and 75% marks in each of the twelve papers
then the candidate should be awarded Distinction.
(iii) If a candidate obtains 60% marks of total marks but fails in maximum three papers in three
different years and marks obtained in these three papers are not below 48% then the candidate
will be entitled for 5 grace marks in each paper, and will be declared pass. And if the candidate

fails in maximum two papers in two different years and marks obtained in these two papers are
not below 46% then the candidate will be entitled for 5 grace marks in each paper and will be
declared pass.
(iv) If a candidate obtains 65% marks of total marks and fails in maximum two papers of two
different years and obtains more than 40% but less than 46% marks in these two papers then the
candidate should be allowed to reappear in the examination for only these two papers.
Following table depicts the marks obtained by some candidates during the course X in three
years. Keeping every aspect in mind, you have to take decision for each case. Give answer.
a) If the candidate is declared pass without grace marks.
b) If the candidate is awarded Distinction.
c) If the candidate is declared pass with grace marks.
d) If the candidate is allowed to reappear for the examination in maximum two papers.
e) If the candidate is declared fail.
(Papers for First

QN

Candid
ate

1.
2.
3.
4.
5.
6.
7.
8.
9.
10.

L
M
N
O
P
Q
R
S
T
U

Papers
I
69
90
76
54
54
50
36
45
90
92

For

Year )

(Papers for Second Year )

First

year

Pap For
ers
II
III
IV
I
II
49
79
81
73
48
82
64
67
82
70
90
83
85
81
84
57
58
57
58
60
56
59
43
58
61
54
52
53
52
54
60
64
68
80
84
50
60
90
95
96
80
70
75
88
87
47
48
88
86
74
*******************************

Sec
ond
III
68
47
75
61
62
56
82
80
86
91

(Papers for Third


Year)

Yea
r
IV
62
80
82
56
66
50
76
80
84
92

Pap
ers
I
49
80
80
56
60
51
75
46
90
88

For
II
70
48
77
62
57
50
80
60
91
86

Thir
d
III
73
76
80
50
55
53
85
65
99
85

Advanced Reasoning (SBI PO level)


Directions (Q. 11-16): In each question below are given two / three statements
followed by two conclusions numbered I and II. You have to take the given statements to be
true even if they seem to be at variance with commonly known facts. Read all the
conclusion and then decide which of the given conclusion logically follows from the given
statements, disregarding commonly known facts. Gove answer
a) if only conclusion I follows
b) if only conclusion II follows.
c) of either conclusion I or II follows
d) if neither conclusion I nor II follows
e) If both conclusions I and II follows.
(11 - 12):

year
IV
77
70
78
56
42
50
90
60
95
80

11.

Statements:

All boats are berries.


No brake is a belt
All belts are boats.
Conclusions: I. No brake is a boat.
II. Some berries are belts.
12.
Conclusions: I. All berries being belts is a possibility
II. All boats being brakes is a possibility.
13.
Statements: Some teas are tasty.
Some toasters are toys
Some tables are not toys.
Conclusions: I. Some toasters are tables.
II. All tea being toasters is a possibility
14.
Statements: All coffee is cold.
Some cold are cups.
Conclusions: I. Some cups being coffee is a possibility
II. All cups being cold is a possibility.
15.
Statements: No rack is a rod
Some rats are red.
All red are racks.
Conclusions: I. Some red are not rods.
II. All rats are racks.
16.
Statements: Some parrots are pigeons
Some pigeons are penguins
No parrot is pink
Conclusions: I. No pigeon is pink
II. Some pink are pigeons.
Directions (Q.17-20): Read the following information carefully and answer the questions
that follows.
There are seven friends - Anita, Binod, Chandni, David, Ejaj, Fazal and Gauri. All of them are
working in an organisation and have different mobile phones - Nokia , Samsung, Spice, Blackberry,
Motorola, Micromax and Videocon - but not necessarily in the same order. Moreover, they are also using
the services of different service providers-Aircel, Vodafone, Reliance, Idea, Tata, Uninorand Airtel.
Further, some more information is also given.
David, a Spice user, does not use Aircel. The one who uses Micromax is a subscriber to Tata.
Anita does not use Vodafone while Binod, who is a Samsung user, is a subscriber to Idea. Ejaj and Fazal
do not use Micromax.
Ejaj, a Reliance user, has Videocon mobile, while the person who uses Blackberry is subscribed to
Vodafone.
Chandni is a Uninor subscriber, while Gauri, who does not Nokia, is an Aircel user.
17.
Who among the following uses Blackberry?
a) Gauri
b) Fazal
c) Chandni
d) Ejaj
e) None of these
18.
Which of the following services is used by Anita?
a) Airtel
b) Aircel
c) Reliance
d) Tata
e) None of these
19.
Which of the following statements is/are definitely false with respect to the above information?
a) Fazal uses Videocon mobile
b) Anita is an Airtel subscriber
c) David is a Reliance subscriber.
d) All are true
e) None of these
20.
Who among the following uses Motorola mobile phones?
a) Anita
b) Gauri
c) Ejaj
d) Fazal
e) Chandni

ANSWERS
11.b

12.a

13.b

14.e

15.a

16.c

17.b

18.d

19.e

20.b

11. b; No brake is a belt (E) i All belts are boats (A) = Ii + A = 0* = Some boats are not brakes. Hence,
conclusion I does not follow. Again, All belts are boats (A) + All boats are berries (A) = A + A = A = All

belts are berries (A) = A + A = A = All belts are berries conversion some berries are belts. Hence,
conclusion II follows.
12.
a; All belts are berries (A). So, there is a possibility that All berries are belts. Hence, conclusion I
follows. Again, Some boats are not brakes. Hence, All boats being brakes is not a possibility. Hence,
conclusion II does not follow.
13.b; I-and O-type statements cant be combined. Hence conclusion I does not follow. Again, since tea is
not related to either table or toy - the only terms that are negatively related - the possibility of II exists.
14. e; Since there is no negative statement, the possibilities of both conclusions I and II exist.
15. a; All red are racks (A) + No rack is a rod (E) = A + E = E = No red is a rod (E) implication Some
red are not rods. Hence conclusion I follows. Some rats are red (I) + All red are racks (A) = I + A = I =
Some rats are racks. Thus, conclusion II also does not follow.
16. c; Some parrots are pigeons conversion Some pigeons are parrots (I) + No parrot is pink (E) = I
+ E = O = Some pigeons are not pink (O*). Hence, neither conclusion I nor conclusion II follows. But
conclusion I and II form a complementary pair (I-E). Thus, either conclusion I or II follows.
(17-20):
Person
Phone
Service Provider
Anita
Micromax
Tata
Binod
Samsung
Idea
Chandni
Nokia
Uninor
David
Spice
Airtel
Ejaj
Videcon
Reliance
Fazal
Blackberry
Vodafone
Gauri
Motorola
Aircel
19. e; All are false

ENGLISH -- useful for SBI PO -Preliminary Exam.


ENGLISH

2.
3.
4.
5.
6.
7.
8.

Directions. Q.1-10: Read each sentence to find out whether there is any error in it. The error, if
any, will be in one part of the sentence. The number of that part is the answer. (Ignore errors of
punctuation, if any).
1.
a) The long gap needs
b) to be bridged soon
c) because it has created
d) a lot of unpleasant
a) He asked the same question
b) that why he was not selected
c) and everyone responded saying
d) that he had failed
a) If they were found
b) guilty during investigation
c) a complaint will be
d) failed
a) They have already put
b) the proposal for to be approved
c) in the ensuing meeting
d) before the Union Cabinet
a)
They had been hired a taxi
b) to roam around the city
c)
but their plan failed
d) as the taxi met with an accident
a) On a number of occasions
b) we had noticed that
c) those two employees
d)
were often in state of confused
a)
Hindi films are certainly
b)
popular in the last decade
c) but in these days our regional films d)
have attained more popularity
a) Most politicians predict
b)
that the same members will

c)

be elected despite of
d)
their failure to perform
9. a) He was selected for the post
b)
because of he was familiar
c) with both urban as well as
d)
rural way of life
10. a) The tribal people have
b) high regard for him because
c) he donated large number of
d) money for their upliftment.
Directions. Q.11 to 15: In each of the following sentences there are two blank spaces. Below
each sentence there are five pairs of words denoted by number (A), (B), (C) and (D). Find out
which paid of words can be filled up in the blanks in the sentence in the same sequence to make
it meaningfully complete.
11. It is ironic that a party, which was once viewed as .. the . of its peasantry, is now its
foe.
a) Persuading, obligations
b) visualizing, sake
c) safeguarding, interests
d) upholding, livelihood
12. Fanaticism of any kind ---------- from any religion is to be -----------a) Emanating, condemned
b) extracted, braved
c) enforced, divorced
d) enshrined,
abandoned
13. The media have done a spectacular job in this regard to -------- their voice and ---- public
opinion.
a)
Curb, garner
b) enrich, expedite c) embolden, unfurl d) raise, sensitise
14. If the Government believes it has done enough to ----- inflation by tinkering with oil
prices, it is -------------a) Derail, unprecedented
b) check, mistaken
c) moderate, foul
d) drive, naive
15. It is high time the Government and private companies ------ more --- to hockey.
a) Cater, prestige
b) deploy, edge
c) paid, attention d) project, tempo
Directions. Q.16-25: In the following passage, there are blanks, each of which has been
numbered. These numbers are printed after the passage and against each, five words are
suggested, one of which fits the blank appropriately. Find out the appropriate word in each case.
Mass migration has produced a huge world wide economy of its own which has . 166 so
fast during the past few years that the figures have 167 experts. Last year remittances
sent home by migrants were expected to ..168 $ 232 billion according to the World Bank
which .. 169 these figures .. 170 .. though the flow of remittances is to alleviate
the plight of the migrants family it cannot on its own lift entire nation out of poverty. Those
who study the .. 171 of remittances argue that the money allows poor countries to put
off basic decisions of economic management like 172 .their tax collection systems and
building schools. Remittances to poor countries can also 173 the fact that they do not
produce much at home. The challenge is now to find programmes that .. 174 the benefits
of remitted cash while 175 . Some of its downside.
16. a) accelerated
b) grew
c) expand
d) increase
17. a) strike
b) encourage c) astonished d) convinced
18. a) rise
b) represent
c) project
d) exceed
19. a) record
b) tracks
c) estimate
d) report
20. a) detrimental
b) minor
c) propose
d) vital
21. a) circumstance b) profit
c) impact
d) status
22. a) declaring
b) established
c) measuring d) reforming
23. a) mark
b) hit
c) review
d) display

24. a) launch
b) predict
c) optimum d) maximise
25. a) augmenting b) avoiding
c) suspend
d) protects
Directions. Q.26-34: Read the following passage carefully and answer the question given below
it. Certain words/ phrases have been printed in bold to help you locate while answering some of
the questions.
Hiero, King of Syracuses, had commissioned from a goldsmith of the town a crown of pure gold,
but, having taken delivery of the finished article, he was suspicious. There was reason to believe
that the craftsman had mixed with the gold a certain amount of other metal of interior value. But
how to find out? There was no direct evidence, and it was therefore obviously a case for the
learned men of the city. And who more learned than Archimedes?
The mathematician was, therefore charged with the task which would nowadays be considered a
simple one, but was then a matter for serious thought. Nothing known to science
thought. Nothing known to science could be brought forward to prove fraud or otherwise on the
part of the goldsmith.
It is more than probable that the human side of the problem interested Archimedes not at all, but
the scientific puzzle worried him intensely. This worry pursued him every where are went for
days., and pre-sisted through the routine acts of his daily round.
In the normal course of that routine, he went to the public baths. We can imagine him standing
at the edge of the bath tub as the prepares to enter it, absently allowing the water to flow until he
cannot help noticing it. Suddenly, he splashed out of his tub, shouting at the top of his voice:
Eureka! Eureka!
Arrived at his house, the mathematician put his newly found discovery to a practical test, and
found indeed that a body plunged in a fluid loses and amount of its weight is equal to the weight
of the fluid displaced by it. With this as a starting point as it was to prove the starting point as it
was to prove the starting point- as it was to prove the starting point to many subsequent
discoveries of importance Archimedes was able to tell his king how much pure gold was in his
crown. Thus, was the first fundamental law in hydrostatics enunciated.
Archimedes was by this time well-known to his fellow townsmen, and his sometimes strange
appearance and unusual actions probably met with indulgent smiles. He came from a good
family; his father Pheidias was an astronomer; he was on intimate terms with, an according to
some was even a kinsman of King Hiero himself.
26. Why could the King not punish the fraudulent gold smith?
a) By that time Archimedes had not discovered the law of hydrostatics.
b) The king did not have concrete evidence to prove the fraud.
c) The finishing of the crown was perfect but deceptive
d) The King had lot of faith in the godsmith.
27. Why was Archimedes charged with the task of finding out if there was any impurity in
the crown?
a) Archimedes was famous as the most learned man and mathematician
b) The King was worried that the goldsmith will tell the truth to Archimedes.
c) The goldsmith was one of the kins of the craftsman.
d) Archimedes was famous for interrogating the criminals and exploring the truth.
28. What was the kings suspicion?
a) The goldsmith had made a crown with some inferior metal instead of gold.
b) The craftsman had replaced gold with a cheaper metal.
c) The goldsmith has mixed a cheaper metal with gold in the crown.

d) The finishing of the crown was not upto the mark.


29. Which of the following statements definitely TRUE in the context of the passage?
a) Archimedes action of running nude through public places was not taken lightly by the
onlookers.
b) Archimedes eccentric actions used to create anguish among the public.
c) The goldsmith and mixed inferior quality metal in the crown.
d) Archimedes could prove that there was no impurity in the gold used by the goldsmith in the
crown.
30. When Archimedes was entrusted with the task, he was curious because
a) he was thrilled by the human side of the problem.
b) He used
31. Eureka! Eureka! as spelt out by Archimedes was
a) An outburst of unreasonable pride of inventing a novel principle.
b) a spontaneous reaction of excitement due to a discovery
c) an immature expression of a half-baked idea
d) an exhibition of encouragement for a useful discovery
32. What was the impact of Archimedes trendous metal involvement in solving the kings
riddle?
a) He was worried but able to take care of his daily routine.
b) The involvement ultimately led to a failure in completing the assignment.
c) he was not able to think of the details of the problem entrusted to him.
d) None of these.
33. A body submerged in a liquid loses its weight equal to
a) the weight of the liquid in which it is submerged.
b) the volume of the liquid in which it is submerged.
c) the weight of the body which is submerged in the liquid.
d) The weight of the liquid displaced by it.
34. Which of the following is FALSE in the context of the passage?
I.
Archimedes father was a relative of King Hiero.
II.
Before Archimedes discovery, science had limitation to detect the goldsmiths fraud.
III.
Archimedes discovered his principle while he was in the tub for bath.
a) I and II b) II and III
c) I and III
d) II only
Directions. Q.35-37 Choose the word/ group of words which is most nearly the SAME in
meaning as the word/group of words given in bold.
35. Enunciated a) Framed
b) Lost
c) Implemented
d) Concocted
36. Charged a) Suspected b) Accused
c) Filled
d) Entrusted
37. Tore out a) Came out gently b) Pushed out c) Walked out d) Rushed out forcefully
Directions. Q.38-40: Choose the word which is most OPPOSITE in meaning of the word given
in bold.
38. Subsequent a) Successive b) Preceding c) Pre-determined d) Consequent
39. Unusual a) Ordinary b) Remarkable c) Abnormal d) Curious
40. Intensely a) Powerfully b) Deeply c) Mildly d) Hugely
Directions. Q.41-45: In the following questions, choose the one which expresses the right
meaning of the given word and mark it the Answer Sheet.
41. Dubious a) Doubtful b) Disputable c) Duplicate d) Dangerous
42. Flabbergasted a) Scared b) Embarrassed c) Dumbfounded d) Humiliated

43. Eternal a) Innumerable b) Unmeasurable c) Prolonged d) Perpetual


44. Genuine a) Authentic b) Legitimate c) Reliable d) Pure
45. Obscene a) Indecent b) Incorrigible c) Ridiculous d) Intolerable
Directions Q. 46-50: In the following questions, out of the four alternatives, choose the one
which can be substituted for the given words/ sentence and mark it in the Answer Sheet.
46. To be biased against
a) Partial
b) Objective c) Prejudiced d) Predestined
47. Motion of head, hands etc., as a mode of expression indicating attitude.
a) Gesture b) Grin c) Gestation d) Grimace
48. Bitter and Violent attack in words
a) Diaspora b) Diacriticism c) Diadem d) Diatribe
49. Treatment by means of exercise and massage
a) Chemotherapy b) Hydrotherapy c) Physiotherapy d) Psychotherapy
50. The abandonment of ones country or cause
a) Defection
b) Disloyalty c) Desertion d) Migration.
ANSWERS
1.
5.

d)
a)

11 c)
21.

2. b)
6. d)

12.

c) 22.

31. b) 32.

41.

3 a) 4. b)
7. a)
8. c) 9.

a) 13.

d)

14.

d) 23. a) 24.

d) 33.

a) 42. c)

d)

43 d)

44.

10. c)

b) 15. c) 16.

d) 25.

34. d)

b)

b) 26. b)

35 a)

a) 45.

a)

a) 17.

c) 18. d)

27 a)

28.

36.

d)37.

c) 38.

46.

c) 47. a)

48.

19 a) 20.

c) 29. c)

b) 39. a)

d) 49.

30.

d)
d)

40. c)

c) 50. a).

Vous aimerez peut-être aussi